Test 7 - Chapters 66-68, 70 & Saunders Neurological

Lakukan tugas rumah & ujian kamu dengan baik sekarang menggunakan Quizwiz!

The nurse is providing instructions to a client who will be taking phenytoin. Which statement, if made by the client, would indicate an understanding of the information about this medication? 1. "I need to perform good oral hygiene, including flossing and brushing my teeth." 2. "I should try to avoid alcohol, but if I'm not able to, I can drink alcohol in moderation." 3. "I should take my medication before coming to the laboratory to have a blood level drawn." 4. "I should monitor for side effects and adjust my medication dose depending on how severe the side effects are."

1. "I need to perform good oral hygiene, including flossing and brushing my teeth."

A client with a spinal cord injury at the level of C5 has a weakened respiratory effort and ineffective cough and is using accessory neck muscles in breathing. The nurse carefully monitors the client and suspects the presence of which problem? 1. Altered breathing pattern 2. Increased likelihood of injury 3. Ineffective oxygen consumption 4. Increased susceptibility to aspiration

1. Altered breathing pattern

The nurse in the neurological unit is monitoring a client for signs of increased intracranial pressure (ICP). The nurse reviews the assessment findings for the client and notes documentation of the presence of Cushing's reflex. The nurse determines that the presence of this reflex is obtained by assessing which item? 1. Blood pressure 2. Motor response 3. Pupillary response 4. Level of consciousness

1. Blood pressure

The nurse is caring for a client after a craniotomy and monitors the client for signs of increased intracranial pressure (ICP). Which finding, if noted in the client, would indicate an early sign of increased ICP? 1. Confusion 2. Bradycardia 3. Sluggish pupils 4. A widened pulse pressure

1. Confusion

The nurse is planning to perform an assessment of the client's level of consciousness using the Glasgow Coma Scale. Which assessments should the nurse include in order to calculate the score? Select all that apply. 1. Eye opening 2. Reflex response 3. Best verbal response 4. Best motor response 5. Pupil size and reaction

1. Eye opening 3. Best verbal response 4.Best motor response

A client with a neurological problem is experiencing hyperthermia. Which measures would be appropriate for the nurse to use in trying to lower the client's body temperature? Select all that apply. 1. Giving tepid sponge baths 2. Applying a hypothermia blanket 3. Covering the client with blankets 4. Administering acetaminophen per protocol 5. Placing ice packs over the client's abdomen and in the axilla and groin

1. Giving tepid sponge baths 2. Applying a hypothermia blanket 4. Administering acetaminophen per protocol

The nurse has just admitted to the nursing unit a client with a basilar skull fracture who is at risk for increased intracranial pressure. Pending specific health care provider prescriptions, the nurse should safely place the client in which positions? Select all that apply. 1. Head midline 2. Neck in neutral position 3. Head of bed elevated 30 to 45 degrees 4. Head turned to the side when flat in bed 5. Neck and jaw flexed forward when opening the mouth

1. Head midline 2. Neck in neutral position 3. Head of bed elevated 30 to 45 degrees

The nurse has just admitted to the nursing unit a client with a basilar skull fracture who is at risk for increased intracranial pressure (ICP). Pending specific health care provider prescriptions, the nurse should plan to place the client in which positions? Select all that apply. 1. Head midline 2. Neck in neutral position 3. Flat, with head turned to the side 4. Head of bed elevated 30 to 45 degrees 5. Head of bed elevated with the neck extended

1. Head midline 2. Neck in neutral position 4. Head of bed elevated 30 to 45 degrees

Members of the family of an unconscious client with increased intracranial pressure are talking at the client's bedside. They are discussing the client's condition and wondering whether the client will ever recover. The nurse intervenes on the basis of which interpretation? 1. It is possible the client can hear the family. 2. The family needs immediate crisis intervention. 3. The client might have wanted a visit from the hospital chaplain. 4. The family could benefit from a conference with the health care provider.

1. It is possible the client can hear the family.

The nurse is caring for a client who begins to experience seizure activity while in bed. Which actions should the nurse take? Select all that apply. 1. Loosening restrictive clothing 2. Restraining the client's limbs 3. Removing the pillow and raising padded side rails 4. Positioning the client to the side, if possible, with the head flexed forward 5. Keeping the curtain around the client and the room door open so when help arrives they can quickly enter to assist

1. Loosening restrictive clothing 3. Removing the pillow and raising padded side rails 4. Positioning the client to the side, if possible, with the head flexed forward

The nurse is instituting seizure precautions for a client who is being admitted from the emergency department. Which measures should the nurse include in planning for the client's safety? Select all that apply. 1. Padding the side rails of the bed 2. Placing an airway at the bedside 3. Placing the bed in the high position 4. Putting a padded tongue blade at the head of the bed 5. Placing oxygen and suction equipment at the bedside 6. Flushing the intravenous catheter to ensure that the site is patent

1. Padding the side rails of the bed 2. Placing an airway at the bedside 5. Placing oxygen and suction equipment at the bedside 6. Flushing the intravenous catheter to ensure that the site is patent

The nurse is assessing a client who is experiencing seizure activity. The nurse understands that it is necessary to determine information about which items as part of routine assessment of seizures? Select all that apply. 1. Postictal status 2. Duration of the seizure 3. Changes in pupil size or eye deviation 4. Seizure progression and type of movements 5. What the client ate in the 2 hours preceding seizure activity

1. Postictal status 2. Duration of the seizure 3. Changes in pupil size or eye deviation 4. Seizure progression and type of movements

The nurse is planning to put aneurysm precautions in place for a client with a cerebral aneurysm. Which nursing measures would be implemented? Select all that apply. 1. Provide physical aspects of care. 2. Prevent pushing or straining activities. 3. Limit caffeinated coffee to 1 cup per day. 4. Keeping the lights on in the client's room. 5. Maintain the head of the bed at 15 degrees.

1. Provide physical aspects of care. 2. Prevent pushing or straining activities. 5. Maintain the head of the bed at 15 degrees.

The nurse is planning care for a client who displays confusion secondary to a neurological problem. Which approaches by the nurse would be helpful in assisting this client? Select all that apply. 1. Providing sensory cues 2. Giving simple, clear directions 3. Providing a stable environment 4. Keeping family pictures at the bedside 5. Encouraging family members to visit at the same time

1. Providing sensory cues 2. Giving simple, clear directions 3. Providing a stable environment 4. Keeping family pictures at the bedside

The nurse is teaching a client with paraplegia measures to maintain skin integrity. Which instruction will be most helpful to the client? 1. Shift weight every 2 hours while in a wheelchair. 2. Change bed sheets every other week to maintain cleanliness. 3. Place a pillow on the seat of the wheelchair to provide extra comfort. 4. Use a mirror to inspect for redness and skin breakdown twice a week.

1. Shift weight every 2 hours while in a wheelchair.

The nurse is trying to communicate with a client who had a stroke and has aphasia. Which actions by the nurse would be most helpful to the client? Select all that apply. 1. Speaking to the client at a slower rate 2. Allowing plenty of time for the client to respond 3. Completing the sentences that the client cannot finish 4. Looking directly at the client during attempts at speech 5. Shouting words if it seems as though the client has difficulty understanding

1. Speaking to the client at a slower rate 2. Allowing plenty of time for the client to respond 4. Looking directly at the client during attempts at speech

A client who had a stroke (brain attack) has right-sided hemianopsia. What should the nurse plan to do to help the client adapt to this problem? 1. Teach the client to scan the environment. 2. Place all objects within the left visual field. 3. Place all objects within the right visual field. 4. Ensure that the family brings the client's eyeglasses to hospital.

1. Teach the client to scan the environment.

The nurse is assigned to care for a client with complete right-sided hemiparesis from a stroke (brain attack). Which characteristics are associated with this condition? Select all that apply. 1. The client is aphasic. 2. The client has weakness on the right side of the body. 3. The client has complete bilateral paralysis of the arms and legs. 4. The client has weakness on the right side of the face and tongue. 5. The client has lost the ability to move the right arm but is able to walk independently. 6. The client has lost the ability to ambulate independently but is able to feed and bathe himself or herself without assistance.

1. The client is aphasic. 2. The client has weakness on the right side of the body. 4. The client has weakness on the right side of the face and tongue.

The nurse is caring for a client with a head injury. The client's intracranial pressure reading is 8 mm Hg. Which condition should the nurse document? 1. The intracranial pressure reading is normal. 2. The intracranial pressure reading is elevated. 3. The intracranial pressure reading is borderline. 4. An intracranial pressure reading of 8 mm Hg is low.

1. The intracranial pressure reading is normal.

The nurse is admitting a client to the hospital emergency department from a nursing home. The client is unconscious with an apparent frontal head injury. A medical diagnosis of epidural hematoma is suspected. Which question is of the highest priority for the emergency department nurse to ask of the transferring nurse at the nursing home? 1. "When did the injury occur?" 2. "Was the client awake and talking right after the injury?" 3. "What medications has the client received since the fall?" 4. "What was the client's level of consciousness before the injury?"

2. "Was the client awake and talking right after the injury?"

The nurse is performing the oculocephalic response (doll's eyes maneuver) on an unconscious client. The nurse turns the client's head and notes movement of the eyes in the same direction as the head. How should the nurse document these findings? 1. Normal 2. Abnormal 3. Insignificant 4. Inconclusive

2. Abnormal

A client who suffered a stroke is prepared for discharge from the hospital. The health care provider has prescribed range-of-motion (ROM) exercises for the client's right side. What action should the nurse include in the client's plan of care? 1. Implement ROM exercises to the point of pain for the client. 2. Consider the use of active, passive, or active-assisted exercises in the home. 3. Encourage the client to be dependent on the home care nurse to complete the exercise program. 4. Develop a schedule of ROM exercises every 2 hours while awake even if the client is fatigued.

2. Consider the use of active, passive, or active-assisted exercises in the home.

The nurse assesses a client who is diagnosed with a stroke (brain attack). On assessment, the client is unable to understand the nurse's commands. Which condition should the nurse document? 1. Occipital lobe impairment 2. Damage to the auditory association areas 3. Frontal lobe and optic nerve tracts damage 4. Difficulty with concept formation and abstraction areas

2. Damage to the auditory association areas

A client is newly admitted to the hospital with a diagnosis of stroke (brain attack) manifested by complete hemiplegia. Which item in the medical history of the client should the nurse be most concerned about? 1. Glaucoma 2. Emphysema 3. Hypertension 4. Diabetes mellitus

2. Emphysema

The nurse is trying to help the family of an unconscious client cope with the situation. Which intervention should the nurse plan to incorporate into the care routine for the client and family? 1. Discouraging the family from touching the client 2. Explaining equipment and procedures on an ongoing basis 3. Ensuring adherence to visiting hours to ensure the client's rest 4. Encouraging the family not to "give in" to their feelings of grief

2. Explaining equipment and procedures on an ongoing basis

The nurse is positioning a client who has increased intracranial pressure. Which position should the nurse avoid? 1. Head midline 2. Head turned to the side 3. Neck in neutral position 4. Head of bed elevated 30 to 45 degrees

2. Head turned to the side

The nurse is caring for a client who is at risk for increased intracranial pressure (ICP) after a stroke. Which activities performed by the nurse will assist with preventing increases in ICP? Select all that apply. 1. Clustering nursing activities 2. Hyperoxygenating before suctioning 3. Maintaining 20 degree flexion of the knees 4. Maintaining the head and neck in midline position 5. Maintaining the head of the bed (HOB) at 30 degrees elevation

2. Hyperoxygenating before suctioning 4. Maintaining the head and neck in midline position 5. Maintaining the head of the bed (HOB) at 30 degrees elevation

The nurse assigned to the care of an unconscious client is making initial daily rounds. On entering the client's room, the nurse observes that the client is lying supine in bed, with the head of the bed elevated approximately 5 degrees. The nasogastric tube feeding is running at 70 mL/hr, as prescribed. The nurse assesses the client and auscultates adventitious breath sounds. Which judgment should the nurse formulate for the client? 1. Impaired nutritional intake 2. Increased risk for aspiration 3. Increased likelihood for injury 4. Susceptibility to fluid volume deficit

2. Increased risk for aspiration

The nurse is caring for the client with increased intracranial pressure. The nurse would note which trend in vital signs if the intracranial pressure is rising? 1. Increasing temperature, increasing pulse, increasing respirations, decreasing blood pressure 2. Increasing temperature, decreasing pulse, decreasing respirations, increasing blood pressure 3. Decreasing temperature, decreasing pulse, increasing respirations, decreasing blood pressure 4. Decreasing temperature, increasing pulse, decreasing respirations, increasing blood pressure

2. Increasing temperature, decreasing pulse, decreasing respirations, increasing blood pressure

A client has a high level of carbon dioxide (CO2) in the bloodstream, as measured by arterial blood gases. The nurse anticipates that which underlying pathophysiology can occur as a result of this elevated CO2? 1. It will cause arteriovenous shunting. 2. It will cause vasodilation of blood vessels in the brain. 3. It will cause blood vessels in the circle of Willis to collapse. 4. It will cause hyperresponsiveness of blood vessels in the brain

2. It will cause vasodilation of blood vessels in the brain.

The home care nurse is making a visit to a client who requires use of a wheelchair after a spinal cord injury sustained 4 months earlier. Just before leaving the home, the nurse ensures that which intervention has been done to prevent an episode of autonomic dysreflexia (hyperreflexia)? 1. Updating the home safety sheet 2. Leaving the client in an unchilled area of the room 3. Noting a bowel movement on the client progress note 4. Recording the amount of urine obtained with catheterization

2. Leaving the client in an unchilled area of the room

The nurse in the health care clinic is providing medication instructions to a client with a seizure disorder who will be taking divalproex sodium. The nurse should instruct the client about the importance of returning to the clinic for monitoring of which laboratory study? 1. Electrolyte panel 2. Liver function studies 3. Renal function studies 4. Blood glucose level determination

2. Liver function studies

A client was seen and treated in the hospital emergency department for a concussion. The nurse determines that the family needs further teaching if they verbalize to call the health care provider (HCP) for which client sign or symptom? 1. Vomiting 2. Minor headache 3. Difficulty speaking 4. Difficulty awakening

2. Minor headache

The nurse is assessing the motor and sensory function of an unconscious client. The nurse should use which technique to test the client's peripheral response to pain? 1. Sternal rub 2. Nail bed pressure 3. Pressure on the orbital rim 4. Squeezing of the sternocleidomastoid muscle

2. Nail bed pressure

The nurse develops a plan of care for a client with a brain aneurysm who will be placed on aneurysm precautions. Which interventions should be included in the plan? Select all that apply. 1. Leave the lights on in the client's room at night. 2. Place a blood pressure cuff at the client's bedside. 3. Close the shades in the client's room during the day. 4. Allow the client to drink 1 cup of caffeinated coffee a day. 5. Allow the client to ambulate 4 times a day with assistance.

2. Place a blood pressure cuff at the client's bedside. 3. Close the shades in the client's room during the day.

The nurse is administering mouth care to an unconscious client. The nurse should perform which actions in the care of this person? Select all that apply. 1. Use products that contain alcohol. 2. Position the client on his or her side. 3. Brush the teeth with a small, soft toothbrush. 4. Cleanse the mucous membranes with soft sponges. 5. Use lemon glycerin swabs when performing mouth care.

2. Position the client on his or her side. 3. Brush the teeth with a small, soft toothbrush. 4. Cleanse the mucous membranes with soft sponges.

At the end of the work shift, the nurse is reviewing the respiratory status of a client admitted with a stroke (brain attack) earlier in the day. The nurse determines that the client's airway is patent if which data are identified? 1. Respiratory rate 24 breaths/min, oxygen saturation 94%, breath sounds clear 2. Respiratory rate 18 breaths/min, oxygen saturation 98%, breath sounds clear 3. Respiratory rate 16 breaths/min, oxygen saturation 85%, wheezes bilaterally 4. Respiratory rate 20 breaths/min, oxygen saturation 92%, diminished breath sounds in lung bases

2. Respiratory rate 18 breaths/min, oxygen saturation 98%, breath sounds clear

The nurse has applied a hypothermia blanket to a client with a fever. The nurse should inspect the skin frequently to detect which condition that is a complication of hypothermia blanket use? 1. Frostbite 2. Skin breakdown 3. Arterial insufficiency 4. Venous insufficiency

2. Skin breakdown

The nurse is documenting nursing observations in the record of a client who experienced a tonic-clonic seizure. Which clinical manifestation did the nurse most likely note in the clonic phase of the seizure? 1. Body stiffening 2. Spasms of the entire body 3. Sudden loss of consciousness 4. Brief flexion of the extremities

2. Spasms of the entire body

A client who has had a stroke (brain attack) has residual dysphagia. When a diet prescription is initiated, the nurse should take which actions? Select all that apply. 1. Giving the client thin liquids 2. Thickening liquids to the consistency of oatmeal 3. Placing food on the unaffected side of the mouth 4. Allowing plenty of time for chewing and swallowing 5. Leave the client alone so that the client will gain independence by feeding self

2. Thickening liquids to the consistency of oatmeal 3. Placing food on the unaffected side of the mouth 4. Allowing plenty of time for chewing and swallowing

The nurse is assisting the neurologist in performing an assessment on a client who is unconscious after sustaining a head injury. The nurse understands that the neurologist would avoid performing the oculocephalic response (doll's eyes maneuver) if which condition is present in the client? 1. Dilated pupils 2. Lumbar trauma 3. A cervical cord injury 4. Altered level of consciousness

3. A cervical cord injury

At 8:00 a.m., A client who has had a stroke (brain attack) was awake and alert with vital signs of temperature 98°F (37.2°C) orally, pulse 80 beats/min, respirations 18 breaths/min, and blood pressure 138/80 mm Hg. At noon, the client is confused and only responsive to tactile stimuli, and vital signs are temperature 99°F (36.7°C) orally, pulse 62 beats/min, respirations 20 breaths/min, and blood pressure 166/72 mm Hg. The nurse should take which action? 1. Reorient the client. 2. Retake the vital signs. 3. Call the health care provider (HCP). 4. Administer an antihypertensive PRN (as needed).

3. Call the health care provider (HCP).

The nurse in the neurological unit is caring for a client who was in a motor vehicle crash and sustained a blunt head injury. On assessment of the client, the nurse notes the presence of bloody drainage from the nose. Which nursing action is most appropriate? 1. Insert nasal packing. 2. Document the findings. 3. Contact the health care provider (HCP). 4. Monitor the client's blood pressure and check for signs of increased intracranial pressure.

3. Contact the health care provider (HCP).

The nurse is assessing a client with a brainstem injury. In addition to obtaining the client's vital signs and determining the Glasgow Coma Scale score, what priority intervention should the nurse plan to implement? 1. Check cranial nerve functioning. 2. Determine the cause of the accident. 3. Draw blood for arterial blood gas analysis. 4. Perform a pulmonary wedge pressure measurement.

3. Draw blood for arterial blood gas analysis.

A client with a neurological impairment experiences urinary incontinence. Which nursing action would be most helpful in assisting the client to adapt to this alteration? 1. Using adult diapers 2. Inserting a Foley catheter 3. Establishing a toileting schedule 4. Padding the bed with an absorbent cotton pad

3. Establishing a toileting schedule

The nurse is evaluating the neurological signs of a client in spinal shock following spinal cord injury. Which observation indicates that spinal shock persists? 1. Hyperreflexia 2. Positive reflexes 3. Flaccid paralysis 4. Reflex emptying of the bladder

3. Flaccid paralysis

The client with a head injury opens eyes to sound, has no verbal response, and localizes to painful stimuli when applied to each extremity. How should the nurse document the Glasgow Coma Scale (GCS) score? 1. GCS = 3 2. GCS = 6 3. GCS = 9 4. GCS = 11

3. GCS = 9

A client with a traumatic closed head injury shows signs of secondary brain injury. What are some manifestations of secondary brain injury? Select all that apply. 1. Fever 2. Seizures 3. Hypoxia 4. Ischemia 5. Hypotension 6. Increased intracranial pressure (ICP)

3. Hypoxia 4. Ischemia 5. Hypotension 6. Increased intracranial pressure (ICP)

The nurse is assessing the nasal dressing on a client who had a transsphenoidal resection of the pituitary gland. The nurse notes a small amount of serosanguineous drainage that is surrounded by clear fluid on the nasal dressing. Which nursing action is most appropriate? 1. Document the findings. 2. Reinforce the dressing. 3. Notify the health care provider (HCP). 4. Mark the area of drainage with a pen and monitor for further drainage.

3. Notify the health care provider (HCP).

The nurse is preparing for the admission to the unit of a client with a diagnosis of seizures and asks the nursing student to institute full seizure precautions. Which item if noted in the client's room would need to be removed and warrants the need to review seizure precautions with the student? 1. Oxygen source 2. Suction machine 3. Padded tongue blade 4. Padding for the side rails

3. Padded tongue blade

The nurse is reviewing the medical records of a client admitted to the nursing unit with a diagnosis of a thrombotic brain attack (stroke). The nurse would expect to note that which is documented in the assessment data section of the record? 1. Sudden loss of consciousness occurred. 2. Signs and symptoms occurred suddenly. 3. The client experienced paresthesias a few days before admission to the hospital. 4. The client complained of a severe headache, which was followed by sudden onset of paralysis.

3. The client experienced paresthesias a few days before admission to the hospital.

To promote optimal cerebral tissue perfusion in the postoperative phase following cranial surgery, the nurse should place the client with an incision in the anterior or middle fossa, in which position? 1. 15 degrees of Trendelenburg's 2. Side-lying with the head of the bed flat 3. With the head of the bed elevated at least 30 degrees 4. With the head of the bed elevated no more than 10 degrees

3. With the head of the bed elevated at least 30 degrees

The nurse is performing an assessment on a client with a diagnosis of thrombotic stroke (brain attack). Which assessment question would elicit data specific to this type of stroke? 1. "Have you had any headaches in the past few days?" 2. "Have you recently been having difficulty with seeing at nighttime?" 3. "Have you had any sudden episodes of passing out in the past few days?" 4. "Have you had any numbness or tingling or paralysis-type feelings in any of your extremities recently?"

4. "Have you had any numbness or tingling or paralysis-type feelings in any of your extremities recently?"

The nurse is instructing a client with Parkinson's disease about preventing falls. Which client statement reflects a need for further teaching? 1. "I can sit down to put on my pants and shoes." 2. "I try to exercise every day and rest when I'm tired." 3. "My son removed all loose rugs from my bedroom." 4. "I don't need to use my walker to get to the bathroom."

4. "I don't need to use my walker to get to the bathroom."

The nurse is caring for a client with an intracranial pressure (ICP) monitoring device. The nurse should become most concerned if the ICP readings drifted to and stayed in the vicinity of which finding? 1. 5 mm Hg 2. 8 mm Hg 3. 14 mm Hg 4. 22 mm Hg

4. 22 mm Hg

A client with a spinal cord injury becomes angry and belligerent whenever the nurse tries to administer care. The nurse should perform which action? 1. Ask the family to deliver the care. 2. Leave the client alone until ready to participate. 3. Advise the client that rehabilitation progresses more quickly with cooperation. 4. Acknowledge the client's anger and continue to encourage participation in care.

4. Acknowledge the client's anger and continue to encourage participation in care.

The nurse is creating a plan of care for a client with a stroke (brain attack) who has global aphasia. The nurse should incorporate communication strategies into the plan of care because of which expected characteristic of the client's speech? 1. Intact 2. Rambling 3. Characterized by literal paraphasia 4. Associated with poor comprehension

4. Associated with poor comprehension

A client who has a spinal cord injury that resulted in paraplegia experiences a sudden onset of severe headache and nausea. The client is diaphoretic with piloerection and has flushing of the skin. The client's systolic blood pressure (BP) is 210 mm Hg. What should the nurse immediately suspect? 1. Return of spinal shock 2. Malignant hypertension 3. Impending brain attack (stroke) 4. Autonomic dysreflexia (hyperreflexia)

4. Autonomic dysreflexia (hyperreflexia)

The nurse is assessing the adaptation of a client to changes in functional status after a stroke (brain attack). Which observation indicates to the nurse that the client is adapting most successfully? 1. Gets angry with family if they interrupt a task 2. Experiences bouts of depression and irritability 3. Has difficulty with using modified feeding utensils 4. Consistently uses adaptive equipment in dressing self

4. Consistently uses adaptive equipment in dressing self

The nurse is teaching a client hospitalized with a seizure disorder and the client's spouse about safety precautions after discharge. The nurse determines that the client needs further teaching if the client states an intention to take which action? 1. Refrain from smoking alone. 2. Take all prescribed medications on time. 3. Have the spouse nearby when showering. 4. Drink alcohol in small amounts and only on weekends.

4. Drink alcohol in small amounts and only on weekends.

A client recovering from a head injury is participating in care. The nurse determines that the client understands measures to prevent elevations in intracranial pressure if the nurse observes the client doing which activity? 1. Blowing the nose 2. Isometric exercises 3. Coughing vigorously 4. Exhaling during repositioning

4. Exhaling during repositioning

A client has clear fluid leaking from the nose following a basilar skull fracture. Which finding would alert the nurse that cerebrospinal fluid is present? 1. Fluid is clear and tests negative for glucose. 2. Fluid is grossly bloody in appearance and has a pH of 6. 3. Fluid clumps together on the dressing and has a pH of 7. 4. Fluid separates into concentric rings and tests positive for glucose.

4. Fluid separates into concentric rings and tests positive for glucose.

The nurse is caring for a client who has just been admitted to the hospital with a diagnosis of a hemorrhagic stroke. The nurse should place the client in which position? 1. Prone 2. Supine 3. Semi Fowler's with the hip and the neck flexed 4. Head of the bed elevated 30 degrees with the head in midline position

4. Head of the bed elevated 30 degrees with the head in midline position

The nurse is creating a plan of care for a client with a diagnosis of stroke (brain attack) with anosognosia. To meet the needs of the client with this deficit, the nurse should include activities that will achieve which outcome? 1. Encourage communication. 2. Provide a consistent daily routine. 3. Promote adequate bowel elimination. 4. Increase the client's awareness of the affected side.

4. Increase the client's awareness of the affected side.

The nurse is creating a plan of care for a client with a stroke (brain attack) who has right homonymous hemianopsia. Which should the nurse include in the plan of care for the client? 1. Place an eye patch on the left eye. 2. Place personal articles on the client's right side. 3. Approach the client from the right field of vision. 4. Instruct the client to turn the head to scan the right visual field.

4. Instruct the client to turn the head to scan the right visual field.

The nurse is caring for a client with a diagnosis of right (nondominant) hemispheric stroke. The nurse notes that the client is alert and oriented to time and place. On the basis of these assessment findings, the nurse should make which interpretation? 1. Had a very mild stroke 2. Most likely suffered a transient ischemic attack 3. May have difficulty with language abilities only 4. Is likely to have perceptual and spatial disabilities

4. Is likely to have perceptual and spatial disabilities

A postoperative craniotomy client who sustained a severe head injury is admitted to the neurological unit. What nursing intervention is necessary for this client? 1. Take and record vital signs every 4 to 8 hours. 2. Prophylactically hyperventilate during the first 24 hours. 3. Treat a central fever with the administration of antipyretic medications such as acetaminophen. 4. Keep the head of the bed elevated at least 30 degrees, and position the client to avoid extreme flexion or extension of the neck and head.

4. Keep the head of the bed elevated at least 30 degrees, and position the client to avoid extreme flexion or extension of the neck and head.

A client with a spinal cord injury expresses little interest in food and is very particular about the choice of meals that are actually eaten. How should the nurse interpret this information? 1. Anorexia is a sign of clinical depression, and a referral to a psychologist is needed. 2. The client has compulsive habits that should be ignored as long as they are not harmful. 3. The client probably has a naturally slow metabolism, and the decreased nutritional intake will not matter. 4. Meal choices represent an area of client control and should be encouraged as much as is nutritionally reasonable.

4. Meal choices represent an area of client control and should be encouraged as much as is nutritionally reasonable.

A client with a traumatic brain injury is on mechanical ventilation. The nurse promotes normal intracranial pressure (ICP) by ensuring that the client's arterial blood gas (ABG) results are within which ranges? 1. PaO2 60 to 100 mm Hg (60 to 100 mm Hg), PaCo2 25 to 30 mm Hg (25 to 30 mm Hg) 2. PaO2 60 to 100 mm Hg (60 to 100 mm Hg), PaCo2 30 to 35 mm Hg (30 to 35 mm Hg) 3. PaO2 80 to 100 mm Hg (80 to 100 mm Hg), PaCo2 25 to 30 mm Hg (25 to 30 mm Hg) 4. PaO2 80 to 100 mm Hg (80 to 100 mm Hg), PaCo2 35 to 38 mm Hg (35 to 38 mm Hg)

4. PaO2 80 to 100 mm Hg (80 to 100 mm Hg), PaCo2 35 to 38 mm Hg (35 to 38 mm Hg)

The nurse is caring for a client who is brought to the hospital emergency department with a spinal cord injury. The nurse minimizes the risk of compounding the injury by performing which action? 1. Keeping the client on a stretcher 2. Logrolling the client onto a soft mattress 3. Logrolling the client onto a firm mattress 4. Placing the client on a bed that provides spinal immobilization

4. Placing the client on a bed that provides spinal immobilization

The nurse has a prescription to begin aneurysm precautions for a client with a subarachnoid hemorrhage secondary to aneurysm rupture. The nurse would plan to incorporate which intervention in controlling the environment for this client? 1. Keep the window blinds open. 2. Turn on a small spotlight above the client's head. 3. Make sure the door to the room is open at all times. 4. Prohibit or limit the use of a radio or television and reading.

4. Prohibit or limit the use of a radio or television and reading.

A client had a transsphenoidal resection of the pituitary gland. The nurse notes drainage on the nasal dressing. Suspecting cerebrospinal fluid (CSF) leakage, the nurse should look for drainage that is of which characteristic? 1. Serosanguineous only 2. Bloody with very small clots 3. Sanguineous only with no clot formation 4. Serosanguineous, surrounded by clear to straw-colored fluid

4. Serosanguineous, surrounded by clear to straw-colored fluid

The nurse is assessing fluid balance in a client who has undergone a craniotomy. The nurse should assess for which finding as a sign of overhydration, which would aggravate cerebral edema? 1. Unchanged weight 2. Shift intake 950 mL, output 900 mL 3. Blood urea nitrogen (BUN) 10 mg/dL (3.6 mmol/L) 4. Serum osmolality 280 mOsm/kg H2O (280 mmol/kg)

4. Serum osmolality 280 mOsm/kg H2O (280 mmol/kg)

The nurse is creating a plan of care for a client with a diagnosis of stroke (brain attack). On reviewing the client's record, the nurse notes an assessment finding of anosognosia. The nursing care plan should address which manifestation related to this finding? 1. The client will be easily fatigued. 2. The client will have difficulty speaking. 3. The client will have difficulty swallowing. 4. The client will exhibit neglect of the affected side.

4. The client will exhibit neglect of the affected side.

The nurse is planning the care of a patient who has been recently diagnosed with a cerebellar tumor. Due to the location of this patients tumor, the nurse should implement measures to prevent what complication? A) Falls B) Audio hallucinations C) Respiratory depression D) Labile BP

Ans: A A cerebellar tumor causes dizziness, an ataxic or staggering gait with a tendency to fall toward the side of the lesion, and marked muscle incoordination. Because of this, the patient faces a high risk of falls. Hallucinations and unstable vital signs are not closely associated with cerebellar tumors.

A patient recovering from a stroke has severe shoulder pain from subluxation of the shoulder and is being cared for on the unit. To prevent further injury and pain, the nurse caring for this patient is aware of what principle of care? A) The patient should be fitted with a cast because use of a sling should be avoided due to adduction of the affected shoulder. B) Elevation of the arm and hand can lead to further complications associated with edema. C) Passively exercising the affected extremity is avoided in order to minimize pain. D) The patient should be taught to interlace fingers, place palms together, and slowly bring scapulae forward to avoid excessive force to shoulder.

Ans: D To prevent shoulder pain, the nurse should never lift a patient by the flaccid shoulder or pull on the affected arm or shoulder. The patient is taught how to move and exercise the affected arm/shoulder through proper movement and positioning. The patient is instructed to interlace the fingers, place the palms together, and push the clasped hands slowly forward to bring the scapulae forward; he or she then raises both hands above the head. This is repeated throughout the day. The use of a properly worn sling when the patient is out of bed prevents the paralyzed upper extremity from dangling without support. Range-of-motion exercises are still vitally important in preventing a frozen shoulder and ultimately atrophy of subcutaneous tissues, which can cause more pain. Elevation of the arm and hand is also important in preventing dependent edema of the hand.

A patient with a head injury has been increasingly agitated and the nurse has consequently identified a risk for injury. What is the nurses best intervention for preventing injury? A) Restrain the patient as ordered. B) Administer opioids PRN as ordered. C) Arrange for friends and family members to sit with the patient. D) Pad the side rails of the patients bed.

Ans: D To protect the patient from self-injury, the nurse uses padded side rails. The nurse should avoid restraints, because straining against them can increase ICP or cause other injury. Narcotics used to control restless patients should be avoided because these medications can depress respiration, constrict the pupils, and alter the patients responsiveness. Visitors should be limited if the patient is agitated.

The nurse is caring for a client who is in the chronic phase of stroke (brain attack) and has a right-sided hemiparesis. The nurse identifies that the client is unable to feed self. Which is the appropriate nursing intervention? 1. Assist the client to eat with the left hand to build strength. 2. Provide a pureed diet that is easy for the client to swallow. 3. Inform the client that a feeding tube will be placed if progress is not made. 4. Provide a variety of foods on the meal tray to stimulate the client's appetite.

1. Assist the client to eat with the left hand to build strength.

The nurse is creating a plan of care for a client with dysphagia following a stroke (brain attack). Which should the nurse include in the plan? Select all that apply. 1. Thicken liquids. 2. Assist the client with eating. 3. Assess for the presence of a swallow reflex. 4. Place the food on the affected side of the mouth. 5. Provide ample time for the client to chew and swallow.

1. Thicken liquids. 2. Assist the client with eating. 3. Assess for the presence of a swallow reflex. 5. Provide ample time for the client to chew and swallow.

The nurse is caring for a client who sustained a spinal cord injury. During administration of morning care, the client begins to exhibit signs and symptoms of autonomic dysreflexia. Which initial nursing action should the nurse take? 1. Elevate the head of the bed. 2. Examine the rectum digitally. 3. Assess the client's blood pressure. 4. Place the client in the prone position.

1. Elevate the head of the bed.

A client who had cranial surgery 5 days earlier to remove a brain tumor has a few cognitive deficits and does not seem to be progressing as quickly as the client or family had hoped. The nurse plans to implement which approach as most helpful to the client and family at this time? 1. Emphasize progress in a realistic manner. 2. Set high goals to give the client something to "aim for." 3. Tell the family to be extremely optimistic with the client. 4. Inform the client and family of standardized goals of care.

1. Emphasize progress in a realistic manner.

The nurse is caring for a client who is on bed rest as part of aneurysm precautions. The nurse should avoid doing which action when giving respiratory care to this client? 1. Encouraging hourly coughing 2. Assisting with incentive spirometer 3. Encouraging hourly deep breathing 4. Repositioning gently side to side every 2 hours

1. Encouraging hourly coughing

The nurse has determined that a client with a neurological disorder also has difficulty breathing. Which activities would be appropriate components of the care plan for this client? Select all that apply. 1. Keep suction equipment at the bedside. 2. Elevate the head of the bed 30 degrees. 3. Keep the client lying in a supine position. 4. Keep the head and neck in good alignment. 5. Administer prescribed respiratory treatments as needed.

1. Keep suction equipment at the bedside. 2. Elevate the head of the bed 30 degrees. 4. Keep the head and neck in good alignment. 5. Administer prescribed respiratory treatments as needed.

A client with a spinal cord injury is prone to experiencing autonomic dysreflexia. The nurse should include which measures in the plan of care to minimize the risk of occurrence? Select all that apply. 1. Keeping the linens wrinkle-free under the client 2. Preventing unnecessary pressure on the lower limbs 3. Limiting bladder catheterization to once every 12 hours 4. Turning and repositioning the client at least every 2 hours 5. Ensuring that the client has a bowel movement at least once a week

1. Keeping the linens wrinkle-free under the client 2. Preventing unnecessary pressure on the lower limbs 4. Turning and repositioning the client at least every 2 hours

The nurse is caring for a client with an intracranial aneurysm who has been alert. Which signs and symptoms are an early indication that the level of consciousness (LOC) is deteriorating? Select all that apply. 1. Mild drowsiness 2. Drooping eyelids 3. Ptosis of the left eyelid 4. Slight slurring of speech 5. Less frequent spontaneous speech

1.Mild drowsiness 4. Slight slurring of speech 5. Less frequent spontaneous speech

The nurse is providing care to a client with increased intracranial pressure (ICP). Which approach is beneficial in controlling the client's ICP from an environmental viewpoint? 1. Reduce environmental noise. 2. Allow visitors as desired by the client and family. 3. Awaken the client every 2 to 3 hours to monitor mental status. 4. Cluster nursing activities to reduce the number of interruptions.

1. Reduce environmental noise.

A client is somewhat nervous about undergoing magnetic resonance imaging (MRI). Which statement by the nurse would provide the most reassurance to the client about the procedure? 1. "The MRI machine is a long, narrow, hollow tube and may make you feel somewhat claustrophobic." 2. "You will be able to eat before the procedure unless you get nauseated easily. If so, you should eat lightly." 3. "Even though you are alone in the scanner, you will be in voice communication with the technologist at all times during the procedure." 4. "It is necessary to remove any metal or metal-containing objects before having the MRI done to avoid the metal being drawn into the magnetic field."

3. "Even though you are alone in the scanner, you will be in voice communication with the technologist at all times during the procedure."

The nurse has instructed the family of a client with stroke (brain attack) who has homonymous hemianopsia about measures to help the client overcome the deficit. Which statement suggests that the family understands the measures to use when caring for the client? 1. "We need to discourage him from wearing eyeglasses." 2. "We need to place objects in his impaired field of vision." 3. "We need to approach him from the impaired field of vision." 4. "We need to remind him to turn his head to scan the lost visual field."

4. "We need to remind him to turn his head to scan the lost visual field."

The nurse is caring for a client with intracranial pressure (ICP) monitoring. Which intervention is appropriate to include in the plan of care? 1. Place the client in Sims' position. 2. Change the drainage tubing every 48 hours. 3. Level the transducer at the lowest point of the ear. 4. Use strict aseptic technique when touching the monitoring system.

4. Use strict aseptic technique when touching the monitoring system.

An older client is brought to the hospital emergency department by a neighbor who heard the client talking and found him wandering in the street at 3 a.m. The nurse should first determine which data about the client? 1. His insurance status 2. Blood toxicology levels 3. Whether he ate his evening meal 4. Whether this is a change in usual level of orientation

4. Whether this is a change in usual level of orientation

A patient is brought by ambulance to the ED after suffering what the family thinks is a stroke. The nurse caring for this patient is aware that an absolute contraindication for thrombolytic therapy is what? A) Evidence of hemorrhagic stroke B) Blood pressure of 180/110 mm Hg C) Evidence of stroke evolution D) Previous thrombolytic therapy within the past 12 months

Ans: A Thrombolytic therapy would exacerbate a hemorrhagic stroke with potentially fatal consequences. Stroke evolution, high BP, or previous thrombolytic therapy does not contraindicate its safe and effective use.

A patient with a C5 spinal cord injury is tetraplegic. After being moved out of the ICU, the patient complains of a severe throbbing headache. What should the nurse do first? A) Check the patients indwelling urinary catheter for kinks to ensure patency. B) Lower the HOB to improve perfusion. C) Administer analgesia. D) Reassure the patient that headaches are expected after spinal cord injuries.

Ans: A A severe throbbing headache is a common symptom of autonomic dysreflexia, which occurs after injuries to the spinal cord above T6. The syndrome is usually brought on by sympathetic stimulation, such as bowel and bladder distention. Lowering the HOB can increase ICP. Before administering analgesia, the nurse should check the patients catheter, record vital signs, and perform an abdominal assessment. A severe throbbing headache is a dangerous symptom in this patient and is not expected.

A patient who just suffered a suspected ischemic stroke is brought to the ED by ambulance. On what should the nurses primary assessment focus? A) Cardiac and respiratory status B) Seizure activity C) Pain D) Fluid and electrolyte balance

Ans: A Acute care begins with managing ABCs. Patients may have difficulty keeping an open and clear airway secondary to decreased LOC. Neurologic assessment with close monitoring for signs of increased neurologic deficit and seizure activity occurs next. Fluid and electrolyte balance must be controlled carefully with the goal of adequate hydration to promote perfusion and decrease further brain activity.

A nurse is assessing a patient with an acoustic neuroma who has been recently admitted to an oncology unit. What symptoms is the nurse likely to find during the initial assessment? A) Loss of hearing, tinnitus, and vertigo B) Loss of vision, change in mental status, and hyperthermia C) Loss of hearing, increased sodium retention, and hypertension D) Loss of vision, headache, and tachycardia

Ans: A An acoustic neuroma is a tumor of the eighth cranial nerve, the cranial nerve most responsible for hearing and balance. The patient with an acoustic neuroma usually experiences loss of hearing, tinnitus, and episodes of vertigo and staggering gait. Acoustic neuromas do not cause loss of vision, increased sodium retention, or tachycardia.

A patient who has experienced an ischemic stroke has been admitted to the medical unit. The patients family in adamant that she remain on bed rest to hasten her recovery and to conserve energy. What principle of care should inform the nurses response to the family? A) The patient should mobilize as soon as she is physically able. B) To prevent contractures and muscle atrophy, bed rest should not exceed 4 weeks. C) The patient should remain on bed rest until she expresses a desire to mobilize. D) Lack of mobility will greatly increase the patients risk of stroke recurrence.

Ans: A As soon as possible, the patient is assisted out of bed and an active rehabilitation program is started. Delaying mobility causes complications, but not necessarily stroke recurrence. Mobility should not be withheld until the patient initiates.

The nurse is caring for a patient whose recent health history includes an altered LOC. What should be the nurses first action when assessing this patient? A) Assessing the patients verbal response B) Assessing the patients ability to follow complex commands C) Assessing the patients judgment D) Assessing the patients response to pain

Ans: A Assessment of the patient with an altered LOC often starts with assessing the verbal response through determining the patients orientation to time, person, and place. In most cases, this assessment will precede each of the other listed assessments, even though each may be indicated.

The school nurse has been called to the football field where player is immobile on the field after landing awkwardly on his head during a play. While awaiting an ambulance, what action should the nurse perform? A) Ensure that the player is not moved. B) Obtain the players vital signs, if possible. C) Perform a rapid assessment of the players range of motion. D) Assess the players reflexes.

Ans: A At the scene of the injury, the patient must be immobilized on a spinal (back) board, with the head and neck maintained in a neutral position, to prevent an incomplete injury from becoming complete. This is a priority over determining the patients vital signs. It would be inappropriate to test ROM or reflexes.

The nurse is caring for a patient whose spinal cord injury has caused recent muscle spasticity. What medication should the nurse expect to be ordered to control this? A) Baclofen (Lioresal) B) Dexamethasone (Decadron) C) Mannitol (Osmitrol) D) Phenobarbital (Luminal)

Ans: A Baclofen is classified as an antispasmodic agent in the treatment of muscles spasms related to spinal cord injury. Decadron is an anti-inflammatory medication used to decrease inflammation in both SCI and head injury. Mannitol is used to decrease cerebral edema in patients with head injury. Phenobarbital is an anticonvulsant that is used in the treatment of seizure activity.

A patient who is being treated in the hospital for a spinal cord injury is advocating for the removal of his urinary catheter, stating that he wants to try to resume normal elimination. What principle should guide the care teams decision regarding this intervention? A) Urinary retention can have serious consequences in patients with SCIs. B) Urinary function is permanently lost following an SCI. C) Urinary catheters should not remain in place for more than 7 days. D) Overuse of urinary catheters can exacerbate nerve damage.

Ans: A Bladder distention, a major cause of autonomic dysreflexia, can also cause trauma. For this reason, removal of a urinary catheter must be considered with caution. Extended use of urinary catheterization is often necessary following SCI. The effect of a spinal cord lesion on urinary function depends on the level of the injury. Catheter use does not cause nerve damage, although it is a major risk factor for UTIs.

A patient is admitted to the neurologic ICU with a suspected diffuse axonal injury. What would be the primary neuroimaging diagnostic tool used on this patient to evaluate the brain structure? A) MRI B) PET scan C) X-ray D) Ultrasound

Ans: A CT and MRI scans, the primary neuroimaging diagnostic tools, are useful in evaluating the brain structure. Ultrasound would not show the brain nor would an x-ray. A PET scan shows brain function, not brain structure.

A patient has developed diabetes insipidus after having increased ICP following head trauma. What nursing assessment best addresses this complication? A) Vigilant monitoring of fluid balance B) Continuous BP monitoring C) Serial arterial blood gases (ABGs) D) Monitoring of the patients airway for patency

Ans: A Diabetes insipidus requires fluid and electrolyte replacement, along with the administration of vasopressin, to replace and slow the urine output. Because of these alterations in fluid balance, careful monitoring is necessary. None of the other listed assessments directly addresses the major manifestations of diabetes insipidus.

A patient with Huntington disease has just been admitted to a long-term care facility. The charge nurse is creating a care plan for this patient. Nutritional management for a patient with Huntington disease should be informed by what principle? A) The patient is likely to have an increased appetite. B) The patient is likely to required enzyme supplements. C) The patient will likely require a clear liquid diet. D) The patient will benefit from a low-protein diet.

Ans: A Due to the continuous involuntary movements, patients will have a ravenous appetite. Despite this ravenous appetite, patients usually become emaciated and exhausted. As the disease progresses, patient experience difficulty in swallowing and thin liquids should be avoided. Protein will not be limited with this disease. Enzyme supplements are not normally required.

A patient with a T2 injury is in spinal shock. The nurse will expect to observe what assessment finding? A) Absence of reflexes along with flaccid extremities B) Positive Babinskis reflex along with spastic extremities C) Hyperreflexia along with spastic extremities D) Spasticity of all four extremities

Ans: A During the period immediately following a spinal cord injury, spinal shock occurs. In spinal shock, all reflexes are absent and the extremities are flaccid. When spinal shock subsides, the patient demonstrates a positive Babinskis reflex, hyperreflexia, and spasticity of all four extremities.

The nurse has created a plan of care for a patient who is at risk for increased ICP. The patients care plan should specify monitoring for what early sign of increased ICP? A) Disorientation and restlessness B) Decreased pulse and respirations C) Projectile vomiting D) Loss of corneal reflex

Ans: A Early indicators of ICP include disorientation and restlessness. Later signs include decreased pulse and respirations, projectile vomiting, and loss of brain stem reflexes, such as the corneal reflex.

When preparing to discharge a patient home, the nurse has met with the family and warned them that the patient may exhibit unexpected emotional responses. The nurse should teach the family that these responses are typically a result of what cause? A) Frustration around changes in function and communication B) Unmet physiologic needs C) Changes in brain activity during sleep and wakefulness D) Temporary changes in metabolism

Ans: A Emotional problems associated with stroke are often related to the new challenges around ADLs and communication. These challenges are more likely than metabolic changes, unmet physiologic needs, or changes in brain activity, each of which should be ruled out.

The nurse is assessing a patient with a suspected stroke. What assessment finding is most suggestive of a stroke? A) Facial droop B) Dysrhythmias C) Periorbital edema D) Projectile vomiting

Ans: A Facial drooping or asymmetry is a classic abnormal finding on a physical assessment that may be associated with a stroke. Facial edema is not suggestive of a stroke and patients less commonly experience dysrhythmias or vomiting.

The nurse is planning the care of a patient with a T1 spinal cord injury. The nurse has identified the diagnosis of risk for impaired skin integrity. How can the nurse best address this risk? A) Change the patients position frequently. B) Provide a high-protein diet. C) Provide light massage at least daily. D) Teach the patient deep breathing and coughing exercises.

Ans: A Frequent position changes are among the best preventative measures against pressure ulcers. A high protein diet can benefit wound healing, but does not necessarily prevent skin breakdown. Light massage and deep breathing do not protect or restore skin integrity.

The nurse is caring for a patient with increased intracranial pressure (ICP) caused by a traumatic brain injury. Which of the following clinical manifestations would suggest that the patient may be experiencing increased brain compression causing brain stem damage? A) Hyperthermia B) Tachycardia C) Hypertension D) Bradypnea

Ans: A Signs of increasing ICP include slowing of the heart rate (bradycardia), increasing systolic BP, and widening pulse pressure. As brain compression increases, respirations become rapid, BP may decrease, and the pulse slows further. A rapid rise in body temperature is regarded as unfavorable. Hyperthermia increases the metabolic demands of the brain and may indicate brain stem damage.

A gerontologic nurse is advocating for diagnostic testing of an 81-year-old patient who is experiencing personality changes. The nurse is aware of what factor that is known to affect the diagnosis and treatment of brain tumors in older adults? A) The effects of brain tumors are often attributed to the cognitive effects of aging. B) Brain tumors in older adults do not normally produce focal effects. C) Older adults typically have numerous benign brain tumors by the eighth decade of life. D) Brain tumors cannot normally be treated in patient over age 75.

Ans: A In older adult patients, early signs and symptoms of intracranial tumors can be easily overlooked or incorrectly attributed to cognitive and neurologic changes associated with normal aging. Brain tumors are not normally benign and they produce focal effects in all patients. Treatment options are not dependent primarily on age.

The nurse is caring for a patient who sustained a moderate head injury following a bicycle accident. The nurses most recent assessment reveals that the patients respiratory effort has increased. What is the nurses most appropriate response? A) Inform the care team and assess for further signs of possible increased ICP. B) Administer bronchodilators as ordered and monitor the patients LOC. C) Increase the patients bed height and reassess in 30 minutes. D) Administer a bolus of normal saline as ordered.

Ans: A Increased respiratory effort can be suggestive of increasing ICP, and the care team should be promptly informed. A bolus of IV fluid will not address the problem. Repositioning the patient and administering bronchodilators are insufficient responses, even though these actions may later be ordered.

After a major ischemic stroke, a possible complication is cerebral edema. Nursing care during the immediate recovery period from an ischemic stroke should include which of the following? A) Positioning to avoid hypoxia B) Maximizing PaCO2 C) Administering hypertonic IV solution D) Initiating early mobilization

Ans: A Interventions during this period include measures to reduce ICP, such as administering an osmotic diuretic (e.g., mannitol), maintaining the partial pressure of carbon dioxide (PaCO2) within the range of 30 to 35 mm Hg, and positioning to avoid hypoxia. Hypertonic IV solutions are not used unless sodium depletion is evident. Mobilization would take place after the immediate threat of increased ICP has past.

A male patient presents at the free clinic with complaints of impotency. Upon physical examination, the nurse practitioner notes the presence of hypogonadism. What diagnosis should the nurse suspect? A) Prolactinoma B) Angioma C) Glioma D) Adrenocorticotropic hormone (ACTH)producing adenoma

Ans: A Male patients with prolactinomas may present with impotence and hypogonadism. An ACTH-producing adenoma would cause acromegaly. The scenario contains insufficient information to know if the tumor is an angioma, glioma, or neuroma.

An older adult has encouraged her husband to visit their primary care provider, stating that she is concerned that he may have Parkinsons disease. Which of the wifes descriptions of her husbands health and function is most suggestive of Parkinsons disease? A) Lately he seems to move far more slowly than he ever has in the past. B) He often complains that his joints are terribly stiff when he wakes up in the morning. C) Hes forgotten the names of some people that weve known for years. D) Hes losing weight even though he has a ravenous appetite.

Ans: A Parkinsons disease is characterized by bradykinesia. It does not manifest as memory loss, increased appetite, or joint stiffness.

A patient with a new diagnosis of amyotrophic lateral sclerosis (ALS) is overwhelmed by his diagnosis and the known complications of the disease. How can the patient best make known his wishes for care as his disease progresses? A) Prepare an advance directive. B) Designate a most responsible physician (MRP) early in the course of the disease. C) Collaborate with representatives from the Amyotrophic Lateral Sclerosis Association. D) Ensure that witnesses are present when he provides instruction.

Ans: A Patients with ALS are encouraged to complete an advance directive or living will to preserve their autonomy in decision making. None of the other listed actions constitutes a legally binding statement of end-of-life care.

A patient who suffered an ischemic stroke now has disturbed sensory perception. What principle should guide the nurses care of this patient? A) The patient should be approached on the side where visual perception is intact. B) Attention to the affected side should be minimized in order to decrease anxiety. C) The patient should avoid turning in the direction of the defective visual field to minimize shoulder subluxation. D) The patient should be approached on the opposite side of where the visual perception is intact to promote recovery.

Ans: A Patients with decreased field of vision should first be approached on the side where visual perception is intact. All visual stimuli should be placed on this side. The patient can and should be taught to turn the head in the direction of the defective visual field to compensate for this loss. The nurse should constantly remind the patient of the other side of the body and should later stand at a position that encourages the patient to move or turn to visualize who and what is in the room.

A patient has been admitted to the neurologic ICU with a diagnosis of a brain tumor. The patient is scheduled to have a tumor resection/removal in the morning. Which of the following assessment parameters should the nurse include in the initial assessment? A) Gag reflex B) Deep tendon reflexes C) Abdominal girth D) Hearing acuity

Ans: A Preoperatively, the gag reflex and ability to swallow are evaluated. In patients with diminished gag response, care includes teaching the patient to direct food and fluids toward the unaffected side, having the patient sit upright to eat, offering a semisoft diet, and having suction readily available. Deep tendon reflexes, abdominal girth, and hearing acuity are less commonly affected by brain tumors and do not affect the risk for aspiration.

A nurse is planning discharge education for a patient who underwent a cervical diskectomy. What strategies would the nurse assess that would aid in planning discharge teaching? A) Care of the cervical collar B) Technique for performing neck ROM exercises C) Home assessment of ABGs D) Techniques for restoring nerve function

Ans: A Prior to discharge, the nurse should assess the patients use and care of the cervical collar. Neck ROM exercises would be contraindicated and ABGs cannot be assessed in the home. Nerve function is not compromised by a diskectomy.

The nurse is caring for a patient who is rapidly progressing toward brain death. The nurse should be aware of what cardinal signs of brain death? Select all that apply. A) Absence of pain response B) Apnea C) Coma D) Absence of brain stem reflexes E) Absence of deep tendon reflexes

Ans: B, C, D The three cardinal signs of brain death upon clinical examination are coma, the absence of brain stem reflexes, and apnea. Absences of pain response and deep tendon reflexes are not necessarily indicative of brain death.

A patient is postoperative day 1 following intracranial surgery. The nurses assessment reveals that the patients LOC is slightly decreased compared with the day of surgery. What is the nurses best response to this assessment finding? A) Recognize that this may represent the peak of post-surgical cerebral edema. B) Alert the surgeon to the possibility of an intracranial hemorrhage. C) Understand that the surgery may have been unsuccessful. D) Recognize the need to refer the patient to the palliative care team.

Ans: A Some degree of cerebral edema occurs after brain surgery; it tends to peak 24 to 36 hours after surgery, producing decreased responsiveness on the second postoperative day. As such, there is not necessarily any need to deem the surgery unsuccessful or to refer the patient to palliative care. A decrease in LOC is not evidence of an intracranial hemorrhage.

The nurse caring for a patient diagnosed with Parkinsons disease has prepared a plan of care that would include what goal? A) Promoting effective communication B) Controlling diarrhea C) Preventing cognitive decline D) Managing choreiform movements

Ans: A The goals for the patient may include improving functional mobility, maintaining independence in ADLs, achieving adequate bowel elimination, attaining and maintaining acceptable nutritional status, achieving effective communication, and developing positive coping mechanisms. Constipation is more likely than diarrhea and cognition largely remains intact. Choreiform movements are related to Huntington disease.

The patient has been diagnosed with aphasia after suffering a stroke. What can the nurse do to best make the patients atmosphere more conducive to communication? A) Provide a board of commonly used needs and phrases. B) Have the patient speak to loved ones on the phone daily. C) Help the patient complete his or her sentences. D) Speak in a loud and deliberate voice to the patient.

Ans: A The inability to talk on the telephone or answer a question or exclusion from conversation causes anger, frustration, fear of the future, and hopelessness. A common pitfall is for the nurse or other health care team member to complete the thoughts or sentences of the patient. This should be avoided because it may cause the patient to feel more frustrated at not being allowed to speak and may deter efforts to practice putting thoughts together and completing a sentence. The patient may also benefit from a communication board, which has pictures of commonly requested needs and phrases. The board may be translated into several languages.

When caring for a patient with increased ICP the nurse knows the importance of monitoring for possible secondary complications, including syndrome of inappropriate antidiuretic hormone (SIADH). What nursing interventions would the nurse most likely initiate if the patient developed SIADH? A) Fluid restriction B) Transfusion of platelets C) Transfusion of fresh frozen plasma (FFP) D) Electrolyte restriction

Ans: A The nurse also assesses for complications of increased ICP, including diabetes insipidus, and SIADH. SIADH requires fluid restriction and monitoring of serum electrolyte levels. Transfusions are unnecessary.

The nurse is caring for a patient with permanent neurologic impairments resulting from a traumatic head injury. When working with this patient and family, what mutual goal should be prioritized? A) Achieve as high a level of function as possible. B) Enhance the quantity of the patients life. C) Teach the family proper care of the patient. D) Provide community assistance.

Ans: A The overarching goals of care are to achieve as high a level of function as possible and to enhance the quality of life for the patient with neurologic impairment and his or her family. This goal encompasses family and community participation.

Following diagnostic testing, a patient has been admitted to the ICU and placed on cerebral aneurysm precautions. What nursing action should be included in patients plan of care? A) Supervise the patients activities of daily living closely. B) Initiate early ambulation to prevent complications of immobility. C) Provide a high-calorie, low-protein diet. D) Perform all of the patients hygiene and feeding.

Ans: A The patient is placed on immediate and absolute bed rest in a quiet, nonstressful environment, because activity, pain, and anxiety elevate BP, which increases the risk for bleeding. As such, independent ADLs and ambulation are contraindicated. There is no need for a high-calorie or low-protein diet.

A patient is recovering from intracranial surgery performed approximately 24 hours ago and is complaining of a headache that the patient rates at 8 on a 10-point pain scale. What nursing action is most appropriate? A) Administer morphine sulfate as ordered. B) Reposition the patient in a prone position. C) Apply a hot pack to the patients scalp. D) Implement distraction techniques.

Ans: A The patient usually has a headache after a craniotomy as a result of stretching and irritation of nerves in the scalp during surgery. Morphine sulfate may also be used in the management of postoperative pain in patients who have undergone a craniotomy. Prone positioning is contraindicated due to the consequent increase in ICP. Distraction would likely be inadequate to reduce pain and a hot pack may cause vasodilation and increased pain.

The nurse is providing health education to a patient who has a C6 spinal cord injury. The patient asks why autonomic dysreflexia is considered an emergency. What would be the nurses best answer? A) The sudden increase in BP can raise the ICP or rupture a cerebral blood vessel. B) The suddenness of the onset of the syndrome tells us the body is struggling to maintain its normal state. C) Autonomic dysreflexia causes permanent damage to delicate nerve fibers that are healing. D) The sudden, severe headache increases muscle tone and can cause further nerve damage.

Ans: A The sudden increase in BP may cause a rupture of one or more cerebral blood vessels or lead to increased ICP. Autonomic dysreflexia does not directly cause nerve damage.

A patient with suspected Parkinsons disease is initially being assessed by the nurse. When is the best time to assess for the presence of a tremor? A) When the patient is resting B) When the patient is ambulating C) When the patient is preparing his or her meal tray to eat D) When the patient is participating in occupational therapy

Ans: A The tremor is present while the patient is at rest; it increases when the patient is walking, concentrating, or feeling anxious. Resting tremor characteristically disappears with purposeful movement, but is evident when the extremities are motionless. Consequently, the nurse should assess for the presence of a tremor when the patient is not performing deliberate actions.

A hospital patient has experienced a seizure. In the immediate recovery period, what action best protects the patients safety? A) Place the patient in a side-lying position. B) Pad the patients bed rails. C) Administer antianxiety medications as ordered. D) Reassure the patient and family members.

Ans: A To prevent complications, the patient is placed in the side-lying position to facilitate drainage of oral secretions. Suctioning is performed, if needed, to maintain a patent airway and prevent aspiration. None of the other listed actions promotes safety during the immediate recovery period.

An 82-year-old man is admitted for observation after a fall. Due to his age, the nurse knows that the patient is at increased risk for what complication of his injury? A) Hematoma B) Skull fracture C) Embolus D) Stroke

Ans: A Two major factors place older adults at increased risk for hematomas. First, the dura becomes more adherent to the skull with increasing age. Second, many older adults take aspirin and anticoagulants as part of routine management of chronic conditions. Because of these factors, the patients risk of a hematoma is likely greater than that of stroke, embolism, or skull fracture.

A patient with spinal cord injury is ready to be discharged home. A family member asks the nurse to review potential complications one more time. What are the potential complications that should be monitored for in this patient? Select all that apply. A) Orthostatic hypotension B) Autonomic dysreflexia C) DVT D) Salt-wasting syndrome E) Increased ICP

Ans: A, B, C For a spinal cord-injured patient, based on the assessment data, potential complications that may develop include DVT, orthostatic hypotension, and autonomic dysreflexia. Salt-wasting syndrome or increased ICP are not typical complications following the immediate recovery period.

The nurse is admitting a patient to the unit who is scheduled for removal of an intracranial mass. What diagnostic procedures might be included in this patients admission orders? Select all that apply. A) Transcranial Doppler flow study B) Cerebral angiography C) MRI D) Cranial radiography E) Electromyelography (EMG)

Ans: A, B, C Preoperative diagnostic procedures may include a CT scan to demonstrate the lesion and show the degree of surrounding brain edema, the ventricular size, and the displacement. An MRI scan provides information similar to that of a CT scan with improved tissue contrast, resolution, and anatomic definition. Cerebral angiography may be used to study a tumors blood supply or to obtain information about vascular lesions. Transcranial Doppler flow studies are used to evaluate the blood flow within intracranial blood vessels. Regular x-rays of the skull would not be diagnostic for an intracranial mass. An EMG would not be ordered prior to intracranial surgery to remove a mass.

A family member of a patient diagnosed with Huntington disease calls you at the clinic. She is requesting help from the Huntingtons Disease Society of America. What kind of help can this patient and family receive from this organization? Select all that apply. A) Information about this disease B) Referrals C) Public education D) Individual assessments E) Appraisals of research studies

Ans: A, B, C The Huntingtons Disease Society of America helps patients and families by providing information, referrals, family and public education, and support for research. It does not provide individual assessments or appraisals of individual research studies.

An adult patient has sought care for the treatment of headaches that have become increasingly severe and frequent over the past several months. Which of the following questions addresses potential etiological factors? Select all that apply? A) Are you exposed to any toxins or chemicals at work? B) How would you describe your ability to cope with stress? C) What medications are you currently taking? D) When was the last time you were hospitalized? E) Does anyone else in your family struggle with headaches?

Ans: A, B, C, E Headaches are multifactorial, and may involve medications, exposure to toxins, family history, and stress. Hospitalization is an unlikely contributor to headaches.

A patient with an inoperable brain tumor has been told that he has a short life expectancy. On what aspects of assessment and care should the home health nurse focus? Select all that apply. A) Pain control B) Management of treatment complications C) Interpretation of diagnostic tests D) Assistance with self-care E) Administration of treatments

Ans: A, B, D, E Home care needs and interventions focus on four major areas: palliation of symptoms and pain control, assistance in self-care, control of treatment complications, and administration of specific forms of treatment, such as parenteral nutrition. Interpretation of diagnostic tests is normally beyond the purview of the nurse.

The nurse caring for a patient in a persistent vegetative state is regularly assessing for potential complications. Complications of neurologic dysfunction for which the nurse should assess include which of the following? Select all that apply. A) Contractures B) Hemorrhage C) Pressure ulcers D) Venous thromboembolism E) Pneumonia

Ans: A, C, D, E Based on the assessment data, potential complications may include respiratory distress or failure, pneumonia, aspiration, pressure ulcer, deep vein thrombosis (DVT), and contractures. The pathophysiology of decreased LOC does not normally create a heightened risk for hemorrhage.

During a patients recovery from stroke, the nurse should be aware of predictors of stroke outcome in order to help patients and families set realistic goals. What are the predictors of stroke outcome? Select all that apply. A) National Institutes of Health Stroke Scale (NIHSS) score B) Race C) LOC at time of admission D) Gender E) Age

Ans: A, C, E It is helpful for clinicians to be knowledgeable about the relative importance of predictors of stroke outcome (age, NIHSS score, and LOC at time of admission) to provide stroke survivors and their families with realistic goals. Race and gender are not predictors of stroke outcome.

The school nurse is giving a presentation on preventing spinal cord injuries (SCI). What should the nurse identify as prominent risk factors for SCI? Select all that apply. A) Young age B) Frequent travel C) African American race D) Male gender E) Alcohol or drug use

Ans: A, D, E The predominant risk factors for SCI include young age, male gender, and alcohol and drug use. Ethnicity and travel are not risk factors.

The nurse is participating in the care of a patient with increased ICP. What diagnostic test is contraindicated in this patients treatment? A) Computed tomography (CT) scan B) Lumbar puncture C) Magnetic resonance imaging (MRI) D) Venous Doppler studies

Ans: B A lumbar puncture in a patient with increased ICP may cause the brain to herniate from the withdrawal of fluid and change in pressure during the lumbar puncture. Herniation of the brain is a dire and frequently fatal event. CT, MRI, and venous Doppler are considered noninvasive procedures and they would not affect the ICP itself.

A nurse is caring for a patient who experiences debilitating cluster headaches. The patient should be taught to take appropriate medications at what point in the course of the onset of a new headache? A) As soon as the patients pain becomes unbearable B) As soon as the patient senses the onset of symptoms C) Twenty to 30 minutes after the onset of symptoms D) When the patient senses his or her symptoms peaking

Ans: B A migraine or a cluster headache in the early phase requires abortive medication therapy instituted as soon as possible. Delaying medication administration would lead to unnecessary pain.

The nurse is caring for a patient diagnosed with Parkinsons disease. The patient is having increasing problems with rising from the sitting to the standing position. What should the nurse suggest to the patient to use that will aid in getting from the sitting to the standing position as well as aid in improving bowel elimination? A) Use of a bedpan B) Use of a raised toilet seat C) Sitting quietly on the toilet every 2 hours D) Following the outlined bowel program

Ans: B A raised toilet seat is useful, because the patient has difficulty in moving from a standing to a sitting position. A handicapped toilet is not high enough and will not aid in improving bowel elimination. Sitting quietly on the toilet every 2 hours will not aid in getting from the sitting to standing position; neither will following the outlined bowel program.

A patient has just returned to the unit from the PACU after surgery for a tumor within the spine. The patient complains of pain. When positioning the patient for comfort and to reduce injury to the surgical site, the nurse will position to patient in what position? A) In the high Fowlers position B) In a flat side-lying position C) In the Trendelenberg position D) In the reverse Trendelenberg position

Ans: B After spinal surgery, the bed is usually kept flat initially. The side-lying position is usually the most comfortable because this position imposes the least pressure on the surgical site. The Fowlers position, Trendelenberg position, and reverse Trendelenberg position are inappropriate for this patient because they would result in increased pain and complications.

A clinic nurse is caring for a patient diagnosed with migraine headaches. During the patient teaching session, the patient questions the nurse regarding alcohol consumption. What would the nurse be correct in telling the patient about the effects of alcohol? A) Alcohol causes hormone fluctuations. B) Alcohol causes vasodilation of the blood vessels. C) Alcohol has an excitatory effect on the CNS. D) Alcohol diminishes endorphins in the brain.

Ans: B Alcohol causes vasodilation of the blood vessels and may exacerbate migraine headaches. Alcohol has a depressant effect on the CNS. Alcohol does not cause hormone fluctuations, nor does it decrease endorphins (morphine-like substances produced by the body) in the brain.

When caring for a patient who had a hemorrhagic stroke, close monitoring of vital signs and neurologic changes is imperative. What is the earliest sign of deterioration in a patient with a hemorrhagic stroke of which the nurse should be aware? A) Generalized pain B) Alteration in level of consciousness (LOC) C) Tonicclonic seizures D) Shortness of breath

Ans: B Alteration in LOC is the earliest sign of deterioration in a patient after a hemorrhagic stroke, such as mild drowsiness, slight slurring of speech, and sluggish papillary reaction. Sudden headache may occur, but generalized pain is less common. Seizures and shortness of breath are not identified as early signs of hemorrhagic stroke.

An elderly woman found with a head injury on the floor of her home is subsequently admitted to the neurologic ICU. What is the best rationale for the following physician orders: elevate the HOB; keep the head in neutral alignment with no neck flexion or head rotation; avoid sharp hip flexion? A) To decrease cerebral arterial pressure B) To avoid impeding venous outflow C) To prevent flexion contractures D) To prevent aspiration of stomach contents

Ans: B Any activity or position that impedes venous outflow from the head may contribute to increased volume inside the skull and possibly increase ICP. Cerebral arterial pressure will be affected by the balance between oxygen and carbon dioxide. Flexion contractures are not a priority at this time. Stomach contents could still be aspirated in this position.

A nurse is caring for a critically ill patient with autonomic dysreflexia. What clinical manifestations would the nurse expect in this patient? A) Respiratory distress and projectile vomiting B) Bradycardia and hypertension C) Tachycardia and agitation D) Third-spacing and hyperthermia

Ans: B Autonomic dysreflexia is characterized by a pounding headache, profuse sweating, nasal congestion, piloerection (goose bumps), bradycardia, and hypertension. It occurs in cord lesions above T6 after spinal shock has resolved; it does not result in vomiting, tachycardia, or third-spacing.

The nurse is caring for a patient who is scheduled for a cervical discectomy the following day. During health education, the patient should be made aware of what potential complications? A) Vertebral fracture B) Hematoma at the surgical site C) Scoliosis D) Renal trauma

Ans: B Based on all the assessment data, the potential complications of diskectomy may include hematoma at the surgical site, resulting in cord compression and neurologic deficit and recurrent or persistent pain after surgery. Renal trauma and fractures are unlikely; scoliosis is a congenital malformation of the spine.

The nurse is caring for a patient recovering from an ischemic stroke. What intervention best addresses a potential complication after an ischemic stroke? A) Providing frequent small meals rather than three larger meals B) Teaching the patient to perform deep breathing and coughing exercises C) Keeping a urinary catheter in situ for the full duration of recovery D) Limiting intake of insoluble fiber

Ans: B Because pneumonia is a potential complication of stroke, deep breathing and coughing exercises should be encouraged unless contraindicated. No particular need exists to provide frequent meals and normally fiber intake should not be restricted. Urinary catheters should be discontinued as soon as possible.

A preceptor is discussing stroke with a new nurse on the unit. The preceptor would tell the new nurse which cardiac dysrhythmia is associated with cardiogenic embolic strokes? A) Ventricular tachycardia B) Atrial fibrillation C) Supraventricular tachycardia D) Bundle branch block

Ans: B Cardiogenic embolic strokes are associated with cardiac dysrhythmias, usually atrial fibrillation. The other listed dysrhythmias are less commonly associated with this type of stroke.

A patient has been admitted to the ICU after being recently diagnosed with an aneurysm and the patients admission orders include specific aneurysm precautions. What nursing action will the nurse incorporate into the patients plan of care? A) Elevate the head of the bed to 45 degrees. B) Maintain the patient on complete bed rest. C) Administer enemas when the patient is constipated. D) Avoid use of thigh-high elastic compression stockings.

Ans: B Cerebral aneurysm precautions are implemented for the patient with a diagnosis of aneurysm to provide a nonstimulating environment, prevent increases in ICP, and prevent further bleeding. The patient is placed on immediate and absolute bed rest in a quiet, nonstressful environment because activity, pain, and anxiety elevate BP, which increases the risk for bleeding. Visitors, except for family, are restricted. The head of the bed is elevated 15 to 30 degrees to promote venous drainage and decrease ICP. Some neurologists, however, prefer that the patient remains flat to increase cerebral perfusion. No enemas are permitted, but stool softeners and mild laxatives are prescribed. Thigh-high elastic compression stockings or sequential compression boots may be ordered to decrease the patients risk for deep vein thrombosis (DVT).

A patient has recently begun mobilizing during the recovery from an ischemic stroke. To protect the patients safety during mobilization, the nurse should perform what action? A) Support the patients full body weight with a waist belt during ambulation. B) Have a colleague follow the patient closely with a wheelchair. C) Avoid mobilizing the patient in the early morning or late evening. D) Ensure that the patients family members do not participate in mobilization.

Ans: B During mobilization, a chair or wheelchair should be readily available in case the patient suddenly becomes fatigued or feels dizzy. The family should be encouraged to participate, as appropriate, and the nurse should not have to support the patients full body weight. Morning and evening activity are not necessarily problematic.

A nurse is caring for a patient diagnosed with a hemorrhagic stroke. When creating this patients plan of care, what goal should be prioritized? A) Prevent complications of immobility. B) Maintain and improve cerebral tissue perfusion. C) Relieve anxiety and pain. D) Relieve sensory deprivation.

Ans: B Each of the listed goals is appropriate in the care of a patient recovering from a stroke. However, promoting cerebral perfusion is a priority physiologic need, on which the patients survival depends.

When caring for a patient who has had a stroke, a priority is reduction of ICP. What patient position is most consistent with this goal? A) Head turned slightly to the right side B) Elevation of the head of the bed C) Position changes every 15 minutes while awake D) Extension of the neck

Ans: B Elevation of the head of the bed promotes venous drainage and lowers ICP; the nurse should avoid flexing or extending the neck or turning the head side to side. The head should be in a neutral midline position. Excessively frequent position changes are unnecessary.

A patient diagnosed with a pituitary adenoma has arrived on the neurologic unit. When planning the patients care, the nurse should be aware that the effects of the tumor will primarily depend on what variable? A) Whether the tumor utilizes aerobic or anaerobic respiration B) The specific hormones secreted by the tumor C) The patients pre-existing health status D) Whether the tumor is primary or the result of metastasis

Ans: B Functioning pituitary tumors can produce one or more hormones normally produced by the anterior pituitary and the effects of the tumor depend largely on the identity of these hormones. This variable is more significant than the patients health status or whether the tumor is primary versus secondary. Anaerobic and aerobic respiration is not relevant.

A patient is brought to the trauma center by ambulance after sustaining a high cervical spinal cord injury 1 hours ago. Endotracheal intubation has been deemed necessary and the nurse is preparing to assist. What nursing diagnosis should the nurse associate with this procedure? A) Risk for impaired skin integrity B) Risk for injury C) Risk for autonomic dysreflexia D) Risk for suffocation

Ans: B If endotracheal intubation is necessary, extreme care is taken to avoid flexing or extending the patients neck, which can result in extension of a cervical injury. Intubation does not directly cause autonomic dysreflexia and the threat to skin integrity is a not a primary concern. Intubation does not carry the potential to cause suffocation.

Following a spinal cord injury a patient is placed in halo traction. While performing pin site care, the nurse notes that one of the traction pins has become detached. The nurse would be correct in implementing what priority nursing action? A) Complete the pin site care to decrease risk of infection. B) Notify the neurosurgeon of the occurrence. C) Stabilize the head in a lateral position. D) Reattach the pin to prevent further head trauma.

Ans: B If one of the pins became detached, the head is stabilized in neutral position by one person while another notifies the neurosurgeon. Reattaching the pin as a nursing intervention would not be done due to risk of increased injury. Pin site care would not be a priority in this instance. Prevention of neurologic injury is the priority.

The nurse in an extended care facility is planning the daily activities of a patient with postpolio syndrome. The nurse recognizes the patient will best benefit from physical therapy when it is scheduled at what time? A) Immediately after meals B) In the morning C) Before bedtime D) In the early evening

Ans: B Important activities for patients with postpolio syndrome should be planned for the morning, as fatigue often increases in the afternoon and evening.

A rehabilitation nurse caring for a patient who has had a stroke is approached by the patients family and asked why the patient has to do so much for herself when she is obviously struggling. What would be the nurses best answer? A) We are trying to help her be as useful as she possibly can. B) The focus on care in a rehabilitation facility is to help the patient to resume as much self-care as possible. C) We arent here to care for her the way the hospital staff did; we are here to help her get better so she can go home. D) Rehabilitation means helping patients do exactly what they did before their stroke.

Ans: B In both acute care and rehabilitation facilities, the focus is on teaching the patient to resume as much self-care as possible. The goal of rehabilitation is not to be useful, nor is it to return patients to their prestroke level of functioning, which may be unrealistic.

While completing a health history on a patient who has recently experienced a seizure, the nurse would assess for what characteristic associated with the postictal state? A) Epileptic cry B) Confusion C) Urinary incontinence D) Body rigidity

Ans: B In the postictal state (after the seizure), the patient is often confused and hard to arouse and may sleep for hours. The epileptic cry occurs from the simultaneous contractions of the diaphragm and chest muscles that occur during the seizure. Urinary incontinence and intense rigidity of the entire body are followed by alternating muscle relaxation and contraction (generalized tonic clonic contraction) during the seizure.

A patient with spinal cord injury has a nursing diagnosis of altered mobility and the nurse recognizes the increased the risk of deep vein thrombosis (DVT). Which of the following would be included as an appropriate nursing intervention to prevent a DVT from occurring? A) Placing the patient on a fluid restriction as ordered B) Applying thigh-high elastic stockings C) Administering an antifibrinolyic agent D) Assisting the patient with passive range of motion (PROM) exercises

Ans: B It is important to promote venous return to the heart and prevent venous stasis in a patient with altered mobility. Applying elastic stockings will aid in the prevention of a DVT. The patient should not be placed on fluid restriction because a dehydrated state will increase the risk of clotting throughout the body. Antifibrinolytic agents cause the blood to clot, which is absolutely contraindicated in this situation. PROM exercises are not an effective protection against the development of DVT.

A patient exhibiting an altered level of consciousness (LOC) due to blunt-force trauma to the head is admitted to the ED. The physician determines the patients injury is causing increased intracranial pressure (ICP). The nurse should gauge the patients LOC on the results of what diagnostic tool? A) Monro-Kellie hypothesis B) Glasgow Coma Scale C) Cranial nerve function D) Mental status examination

Ans: B LOC, a sensitive indicator of neurologic function, is assessed based on the criteria in the Glasgow Coma Scale: eye opening, verbal response, and motor response. The Monro-Kellie hypothesis states that because of the limited space for expansion within the skull, an increase in any one of the components (blood, brain tissue, cerebrospinal fluid) causes a change in the volume of the others. Cranial nerve function and the mental status examination would be part of the neurologic examination for this patient, but would not be the priority in evaluating LOC.

The nurse is providing care for a patient who is unconscious. What nursing intervention takes highest priority? A) Maintaining accurate records of intake and output B) Maintaining a patent airway C) Inserting a nasogastric (NG) tube as ordered D) Providing appropriate pain control

Ans: B Maintaining a patent airway always takes top priority, even though each of the other listed actions is necessary and appropriate.

The staff educator is precepting a nurse new to the critical care unit when a patient with a T2 spinal cord injury is admitted. The patient is soon exhibiting manifestations of neurogenic shock. In addition to monitoring the patient closely, what would be the nurses most appropriate action? A) Prepare to transfuse packed red blood cells. B) Prepare for interventions to increase the patients BP. C) Place the patient in the Trendelenberg position. D) Prepare an ice bath to lower core body temperature.

Ans: B Manifestations of neurogenic shock include decreased BP and heart rate. Cardiac markers would be expected to rise in cardiogenic shock. Transfusion, repositioning, and ice baths are not indicated interventions.

The nurse is caring for a patient who is in status epilepticus. What medication does the nurse know may be given to halt the seizure immediately? A) Intravenous phenobarbital (Luminal) B) Intravenous diazepam (Valium) C) Oral lorazepam (Ativan) D) Oral phenytoin (Dilantin)

Ans: B Medical management of status epilepticus includes IV diazepam (Valium) and IV lorazepam (Ativan) given slowly in an attempt to halt seizures immediately. Other medications (phenytoin, phenobarbital) are given later to maintain a seizure-free state. Oral medications are not given during status epilepticus.

A neurologic flow chart is often used to document the care of a patient with a traumatic brain injury. At what point in the patients care should the nurse begin to use a neurologic flow chart? A) When the patients condition begins to deteriorate B) As soon as the initial assessment is made C) At the beginning of each shift D) When there is a clinically significant change in the patients condition

Ans: B Neurologic parameters are assessed initially and as frequently as the patients condition requires. As soon as the initial assessment is made, the use of a neurologic flowchart is started and maintained. A new chart is not begun at the start of every shift.

A patient who has sustained a nondepressed skull fracture is admitted to the acute medical unit. Nursing care should include which of the following? A) Preparation for emergency craniotomy B) Watchful waiting and close monitoring C) Administration of inotropic drugs D) Fluid resuscitation

Ans: B Nondepressed skull fractures generally do not require surgical treatment; however, close observation of the patient is essential. A craniotomy would not likely be needed if the fracture is nondepressed. Even if treatment is warranted, it is unlikely to include inotropes or fluid resuscitation.

A 25-year-old female patient with brain metastases is considering her life expectancy after her most recent meeting with her oncologist. Based on the fact that the patient is not receiving treatment for her brain metastases, what is the nurses most appropriate action? A) Promoting the patients functional status and ADLs B) Ensuring that the patient receives adequate palliative care C) Ensuring that the family does not tell the patient that her condition is terminal D) Promoting adherence to the prescribed medication regimen

Ans: B Patients with intracerebral metastases who are not treated have a steady downhill course with a limited survival time, whereas those who are treated may survive for slightly longer periods, but for most cure is not possible. Palliative care is thus necessary. This is a priority over promotion of function and the family should not normally withhold information from the patient. Adherence to medications such as analgesics is important, but palliative care is a high priority.

A patient with increased ICP has a ventriculostomy for monitoring ICP. The nurses most recent assessment reveals that the patient is now exhibiting nuchal rigidity and photophobia. The nurse would be correct in suspecting the presence of what complication? A) Encephalitis B) CSF leak C) Meningitis D) Catheter occlusion

Ans: C Complications of a ventriculostomy include ventricular infectious meningitis and problems with the monitoring system. Nuchal rigidity and photophobia are clinical manifestations of meningitis, but are not suggestive of encephalitis, a CSF leak, or an occluded catheter.

A patient has been admitted to the neurologic unit for the treatment of a newly diagnosed brain tumor. The patient has just exhibited seizure activity for the first time. What is the nurses priority response to this event? A) Identify the triggers that precipitated the seizure. B) Implement precautions to ensure the patients safety. C) Teach the patients family about the relationship between brain tumors and seizure activity. D) Ensure that the patient is housed in a private room.

Ans: B Patients with seizures are carefully monitored and protected from injury. Patient safety is a priority over health education, even though this is appropriate and necessary. Specific triggers may or may not be evident; identifying these is not the highest priority. A private room is preferable, but not absolutely necessary.

The nurse is caring for a boy who has muscular dystrophy. When planning assistance with the patients ADLs, what goal should the nurse prioritize? A) Promoting the patients recovery from the disease B) Maximizing the patients level of function C) Ensuring the patients adherence to treatment D) Fostering the familys participation in care

Ans: B Priority for the care of the child with muscular dystrophy is the need to maximize the patients level of function. Family participation is also important, but should be guided by this goal. Adherence is not a central goal, even though it is highly beneficial, and the disease is not curable.

A patient newly diagnosed with a cervical disk herniation is receiving health education from the clinic nurse. What conservative management measures should the nurse teach the patient to implement? A) Perform active ROM exercises three times daily. B) Sleep on a firm mattress. C) Apply cool compresses to the back of the neck daily. D) Wear the cervical collar for at least 2 hours at a time.

Ans: B Proper positioning on a firm mattress and bed rest for 1 to 2 days may bring dramatic relief from pain. The patient may need to wear a cervical collar 24 hours a day during the acute phase of pain from a cervical disk herniation. Hot, moist compresses applied to the back of the neck will increase blood flow to the muscles and help relax the spastic muscles.

A family member brings the patient to the clinic for a follow-up visit after a stroke. The family member asks the nurse what he can do to decrease his chance of having another stroke. What would be the nurses best answer? A) Have your heart checked regularly. B) Stop smoking as soon as possible. C) Get medication to bring down your sodium levels. D) Eat a nutritious diet.

Ans: B Smoking is a modifiable and highly significant risk factor for stroke. The significance of smoking, and the potential benefits of quitting, exceed the roles of sodium, diet, and regular medical assessments.

A school nurse is called to the playground where a 6-year-old girl has been found unresponsive and staring into space, according to the playground supervisor. How would the nurse document the girls activity in her chart at school? A) Generalized seizure B) Absence seizure C) Focal seizure D) Unclassified seizure

Ans: B Staring episodes characterize an absence seizure, whereas focal seizures, generalized seizures, and unclassified seizures involve uncontrolled motor activity.

A nurse is reviewing the trend of a patients scores on the Glasgow Coma Scale (GCS). This allows the nurse to gauge what aspect of the patients status? A) Reflex activity B) Level of consciousness C) Cognitive ability D) Sensory involvement

Ans: B The Glasgow Coma Scale (GCS) examines three responses related to LOC: eye opening, best verbal response, and best motor response.

The neurologic ICU nurse is admitting a patient following a craniotomy using the supratentorial approach. How should the nurse best position the patient? A) Position the patient supine. B) Maintain head of bed (HOB) elevated at 30 to 45 degrees. C) Position patient in prone position. D) Maintain bed in Trendelenberg position.

Ans: B The patient undergoing a craniotomy with a supratentorial (above the tentorium) approach should be placed with the HOB elevated 30 to 45 degrees, with the neck in neutral alignment. Each of the other listed positions would cause a dangerous elevation in ICP.

The nurse is caring for a patient who is postoperative following a craniotomy. When writing the plan of care, the nurse identifies a diagnosis of deficient fluid volume related to fluid restriction and osmotic diuretic use. What would be an appropriate intervention for this diagnosis? A) Change the patients position as indicated. B) Monitor serum electrolytes. C) Maintain NPO status. D) Monitor arterial blood gas (ABG) values.

Ans: B The postoperative fluid regimen depends on the type of neurosurgical procedure and is determined on an individual basis. The volume and composition of fluids are adjusted based on daily serum electrolyte values, along with fluid intake and output. Fluids may have to be restricted in patients with cerebral edema. Changing the patients position, maintaining an NPO status, and monitoring ABG values do not relate to the nursing diagnosis of deficient fluid volume.

The nurse is performing stroke risk screenings at a hospital open house. The nurse has identified four patients who might be at risk for a stroke. Which patient is likely at the highest risk for a hemorrhagic stroke? A) White female, age 60, with history of excessive alcohol intake B) White male, age 60, with history of uncontrolled hypertension C) Black male, age 60, with history of diabetes D) Black male, age 50, with history of smoking

Ans: B Uncontrolled hypertension is the primary cause of a hemorrhagic stroke. Control of hypertension, especially in individuals over 55 years of age, clearly reduces the risk for hemorrhagic stroke. Additional risk factors are increased age, male gender, and excessive alcohol intake. Another high-risk group includes African Americans, where the incidence of first stroke is almost twice that as in Caucasians.

The clinic nurse caring for a patient with Parkinsons disease notes that the patient has been taking levodopa and carbidopa (Sinemet) for 7 years. For what common side effect of Sinemet would the nurse assesses this patient? A) Pruritus B) Dyskinesia C) Lactose intolerance D) Diarrhea

Ans: B Within 5 to 10 years of taking levodopa, most patients develop a response to the medication characterized by dyskinesia (abnormal involuntary movements). Another potential complication of longterm dopaminergic medication use is neuroleptic malignant syndrome characterized by severe rigidity, stupor, and hyperthermia. Side effects of long-term Sinemet therapy are not pruritus, lactose intolerance, or diarrhea.

As a member of the stroke team, the nurse knows that thrombolytic therapy carries the potential for benefit and for harm. The nurse should be cognizant of what contraindications for thrombolytic therapy? Select all that apply. A) INR above 1.0 B) Recent intracranial pathology C) Sudden symptom onset D) Current anticoagulation therapy E) Symptom onset greater than 3 hours prior to admission

Ans: B, D, E Some of the absolute contraindications for thrombolytic therapy include symptom onset greater than 3 hours before admission, a patient who is anticoagulated (with an INR above 1.7), or a patient who has recently had any type of intracranial pathology (e.g., previous stroke, head injury, trauma).

A patient is admitted to the neurologic ICU with a C4 spinal cord injury. When writing the plan of care for this patient, which of the following nursing diagnoses would the nurse prioritize in the immediate care of this patient? A) Risk for impaired skin integrity related to immobility and sensory loss B) Impaired physical mobility related to loss of motor function C) Ineffective breathing patterns related to weakness of the intercostal muscles D) Urinary retention related to inability to void spontaneously

Ans: C A nursing diagnosis related to breathing pattern would be the priority for this patient. A C4 spinal cord injury will require ventilatory support, due to the diaphragm and intercostals being affected. The other nursing diagnoses would be used in the care plan, but not designated as a higher priority than ineffective breathing patterns.

A patient has had an ischemic stroke and has been admitted to the medical unit. What action should the nurse perform to best prevent joint deformities? A) Place the patient in the prone position for 30 minutes/day. B) Assist the patient in acutely flexing the thigh to promote movement. C) Place a pillow in the axilla when there is limited external rotation. D) Place patients hand in pronation.

Ans: C A pillow in the axilla prevents adduction of the affected shoulder and keeps the arm away from the chest. The prone position with a pillow under the pelvis, not flat, promotes hyperextension of the hip joints, essential for normal gait. To promote venous return and prevent edema, the upper thigh should not be flexed acutely. The hand is placed in slight supination, not pronation, which is its most functional position.

A patient with Parkinsons disease is undergoing a swallowing assessment because she has recently developed adventitious lung sounds. The patients nutritional needs should be met by what method? A) Total parenteral nutrition (TPN) B) Provision of a low-residue diet C) Semisolid food with thick liquids D) Minced foods and a fluid restriction

Ans: C A semisolid diet with thick liquids is easier for a patient with swallowing difficulties to consume than is a solid diet. Low-residue foods and fluid restriction are unnecessary and counterproductive to the patients nutritional status. The patients status does not warrant TPN.

A community health nurse is giving an educational presentation about stroke and heart disease at the local senior citizens center. What nonmodifiable risk factor for stroke should the nurse cite? A) Female gender B) Asian American race C) Advanced age D) Smoking

Ans: C Advanced age, male gender, and race are well-known nonmodifiable risk factors for stroke. High-risk groups include people older than 55 years of age; the incidence of stroke more than doubles in each successive decade. Men have a higher rate of stroke than that of women. Another high-risk group is African Americans; the incidence of first stroke in African Americans is almost twice that as in Caucasian Americans; Asian American race is not a risk factor. Smoking is a modifiable risk.

The ED nurse is caring for a patient who has been brought in by ambulance after sustaining a fall at home. What physical assessment finding is suggestive of a basilar skull fracture? A) Epistaxis B) Periorbital edema C) Bruising over the mastoid D) Unilateral facial numbness

Ans: C An area of ecchymosis (bruising) may be seen over the mastoid (Battles sign) in a basilar skull fracture. Numbness, edema, and epistaxis are not directly associated with a basilar skull fracture.

A patient is brought to the ED by her family after falling off the roof. A family member tells the nurse that when the patient fell she was knocked out, but came to and seemed okay. Now she is complaining of a severe headache and not feeling well. The care team suspects an epidural hematoma, prompting the nurse to prepare for which priority intervention? A) Insertion of an intracranial monitoring device B) Treatment with antihypertensives C) Emergency craniotomy D) Administration of anticoagulant therapy

Ans: C An epidural hematoma is considered an extreme emergency. Marked neurologic deficit or respiratory arrest can occur within minutes. Treatment consists of making an opening through the skull to decrease ICP emergently, remove the clot, and control the bleeding. Antihypertensive medications would not be a priority. Anticoagulant therapy should not be ordered for a patient who has a cranial bleed. This could further increase bleeding activity. Insertion of an intracranial monitoring device may be done during the surgery, but is not priority for this patient.

A patient with a new diagnosis of ischemic stroke is deemed to be a candidate for treatment with tissue plasminogen activator (t-PA) and has been admitted to the ICU. In addition to closely monitoring the patients cardiac and neurologic status, the nurse monitors the patient for signs of what complication? A) Acute pain B) Septicemia C) Bleeding D) Seizures

Ans: C Bleeding is the most common side effect of t-PA administration, and the patient is closely monitored for any bleeding. Septicemia, pain, and seizures are much less likely to result from thrombolytic therapy.

A patient with a brain tumor has begun to exhibit signs of cachexia. What subsequent assessment should the nurse prioritize? A) Assessment of peripheral nervous function B) Assessment of cranial nerve function C) Assessment of nutritional status D) Assessment of respiratory status

Ans: C Cachexia is a wasting syndrome of weight loss, muscle atrophy, fatigue, weakness, and significant loss of appetite. Consequently, nutritional assessment is paramount.

Following a traumatic brain injury, a patient has been in a coma for several days. Which of the following statements is true of this patients current LOC? A) The patient occasionally makes incomprehensible sounds. B) The patients current LOC will likely become a permanent state. C) The patient may occasionally make nonpurposeful movements. D) The patient is incapable of spontaneous respirations.

Ans: C Coma is a clinical state of unarousable unresponsiveness in which no purposeful responses to internal or external stimuli occur, although nonpurposeful responses to painful stimuli and brain stem reflexes may be present. Verbal sounds, however, are atypical. Ventilator support may or may not be necessary. Comas are not permanent states.

The nurse is discharging home a patient who suffered a stroke. He has a flaccid right arm and leg and is experiencing problems with urinary incontinence. The nurse makes a referral to a home health nurse because of an awareness of what common patient response to a change in body image? A) Denial B) Fear C) Depression D) Disassociation

Ans: C Depression is a common and serious problem in the patient who has had a stroke. It can result from a profound disruption in his or her life and changes in total function, leaving the patient with a loss of independence. The nurse needs to encourage the patient to verbalize feelings to assess the effect of the stroke on self-esteem. Denial, fear, and disassociation are not the most common patient response to a change in body image, although each can occur in some patients.

A patient diagnosed with a hemorrhagic stroke has been admitted to the neurologic ICU. The nurse knows that teaching for the patient and family needs to begin as soon as the patient is settled on the unit and will continue until the patient is discharged. What will family education need to include? A) How to differentiate between hemorrhagic and ischemic stroke B) Risk factors for ischemic stroke C) How to correctly modify the home environment D) Techniques for adjusting the patients medication dosages at home

Ans: C For a patient with a hemorrhagic stroke, teaching addresses the use of assistive devices or modification of the home environment to help the patient live with the disability. This is more important to the patients needs than knowing about risk factors for ischemic stroke. It is not necessary for the family to differentiate between different types of strokes. Medication regimens should never be altered without consultation.

A patient has experienced a seizure in which she became rigid and then experienced alternating muscle relaxation and contraction. What type of seizure does the nurse recognize? A) Unclassified seizure B) Absence seizure C) Generalized seizure D) Focal seizure

Ans: C Generalized seizures often involve both hemispheres of the brain, causing both sides of the body to react. Intense rigidity of the entire body may occur, followed by alternating muscle relaxation and contraction (generalized tonicclonic contraction). This pattern of rigidity does not occur in patients who experience unclassified, absence, or focal seizures.

A patient who has been on long-term phenytoin (Dilantin) therapy is admitted to the unit. In light of the adverse of effects of this medication, the nurse should prioritize which of the following in the patients plan of care? A) Monitoring of pulse oximetry B) Administration of a low-protein diet C) Administration of thorough oral hygiene D) Fluid restriction as ordered

Ans: C Gingival hyperplasia (swollen and tender gums) can be associated with long-term phenytoin (Dilantin) use. Thorough oral hygiene should be provided consistently and encouraged after discharge. Fluid and protein restriction are contraindicated and there is no particular need for constant oxygen saturation monitoring.

A patient, brought to the clinic by his wife and son, is diagnosed with Huntington disease. When providing anticipatory guidance, the nurse should address the future possibility of what effect of Huntington disease? A) Metastasis B) Risk for stroke C) Emotional and personality changes D) Pathologic bone fractures

Ans: C Huntington disease causes profound changes to personality and behavior. It is a nonmalignant disease and stroke is not a central risk. The disease is not associated with pathologic bone fractures.

The nurse is caring for a patient with a brain tumor. What drug would the nurse expect to be ordered to reduce the edema surrounding the tumor? A) Solumedrol B) Dextromethorphan C) Dexamethasone D) Furosemide

Ans: C If a brain tumor is the cause of the increased ICP, corticosteroids (e.g., dexamethasone) help reduce the edema surrounding the tumor. Solumedrol, a steroid, and furosemide, a loop diuretic, are not the drugs of choice in this instance. Dextromethorphan is used in cough medicines.

The nurse is caring for a patient diagnosed with an ischemic stroke and knows that effective positioning of the patient is important. Which of the following should be integrated into the patients plan of care? A) The patients hip joint should be maintained in a flexed position. B) The patient should be in a supine position unless ambulating. C) The patient should be placed in a prone position for 15 to 30 minutes several times a day. D) The patient should be placed in a Trendelenberg position two to three times daily to promote cerebral perfusion

Ans: C If possible, the patient is placed in a prone position for 15 to 30 minutes several times a day. A small pillow or a support is placed under the pelvis, extending from the level of the umbilicus to the upper third of the thigh. This helps to promote hyperextension of the hip joints, which is essential for normal gait, and helps prevent knee and hip flexion contractures. The hip joints should not be maintained in flexion and the Trendelenberg position is not indicated.

The nurse responds to the call light of a patient who has had a cervical diskectomy earlier in the day. The patient states that she is having severe pain that had a sudden onset. What is the nurses most appropriate action? A) Palpate the surgical site. B) Remove the dressing to assess the surgical site. C) Call the surgeon to report the patients pain. D) Administer a dose of an NSAID.

Ans: C If the patient experiences a sudden increase in pain, extrusion of the graft may have occurred, requiring reoperation. A sudden increase in pain should be promptly reported to the surgeon. Administration of an NSAID would be an insufficient response and the dressing should not be removed without an order. Palpation could cause further damage.

Paramedics have brought an intubated patient to the RD following a head injury due to accelerationdeceleration motor vehicle accident. Increased ICP is suspected. Appropriate nursing interventions would include which of the following? A) Keep the head of the bed (HOB) flat at all times. B) Teach the patient to perform the Valsalva maneuver. C) Administer benzodiazepines on a PRN basis. D) Perform endotracheal suctioning every hour.

Ans: C If the patient with a brain injury is very agitated, benzodiazepines are the most commonly used sedatives and do not affect cerebral blood flow or ICP. The HOB should be elevated 30 degrees. Suctioning should be done a limited basis, due to increasing the pressure in the cranium. The Valsalva maneuver is to be avoided. This also causes increased ICP.

A nurse is admitting a patient with a severe migraine headache and a history of acute coronary syndrome. What migraine medication would the nurse question for this patient? A) Rizatriptan (Maxalt) B) Naratriptan (Amerge) C) Sumatriptan succinate (Imitrex) D) Zolmitriptan (Zomig)

Ans: C Triptans can cause chest pain and are contraindicated in patients with ischemic heart disease. Maxalt, Amerge, and Zomig are triptans used in routine clinical use for the treatment of migraine headaches.

A patient with amyotrophic lateral sclerosis (ALS) is being visited by the home health nurse who is creating a care plan. What nursing diagnosis is most likely for a patient with this condition? A) Chronic confusion B) Impaired urinary elimination C) Impaired verbal communication D) Bowel incontinence

Ans: C Impaired communication is an appropriate nursing diagnosis; the voice in patients with ALS assumes a nasal sound and articulation becomes so disrupted that speech is unintelligible. Intellectual function is marginally impaired in patients with late ALS. Usually, the anal and bladder sphincters are intact because the spinal nerves that control muscles of the rectum and urinary bladder are not affected.

A patient is admitted to the neurologic ICU with a spinal cord injury. When assessing the patient the nurse notes there is a sudden depression of reflex activity in the spinal cord below the level of injury. What should the nurse suspect? A) Epidural hemorrhage B) Hypertensive emergency C) Spinal shock D) Hypovolemia

Ans: C In spinal shock, the reflexes are absent, BP and heart rate fall, and respiratory failure can occur. Hypovolemia, hemorrhage, and hypertension do not cause this sudden change in neurologic function.

A patient with Parkinsons disease is experiencing episodes of constipation that are becoming increasingly frequent and severe. The patient states that he has been achieving relief for the past few weeks by using OTC laxatives. How should the nurse respond? A) Its important to drink plenty of fluids while youre taking laxatives. B) Make sure that you supplement your laxatives with a nutritious diet. C) Lets explore other options, because laxatives can have side effects and create dependency. D) You should ideally be using herbal remedies rather than medications to promote bowel function.

Ans: C Laxatives should be avoided in patients with Parkinsons disease due to the risk of adverse effects and dependence. Herbal bowel remedies are not necessarily less risky.

The nurse recognizes that a patient with a SCI is at risk for muscle spasticity. How can the nurse best prevent this complication of an SCI? A) Position the patient in a high Fowlers position when in bed. B) Support the knees with a pillow when the patient is in bed. C) Perform passive ROM exercises as ordered. D) Administer NSAIDs as ordered.

Ans: C Passive ROM exercises can prevent muscle spasticity following SCI. NSAIDs are not used for this purpose. Pillows and sitting upright do not directly address the patients risk of muscle spasticity.

A patient is admitted to the neurologic ICU with a spinal cord injury. In writing the patients care plan, the nurse specifies that contractures can best be prevented by what action? A) Repositioning the patient every 2 hours B) Initiating range-of-motion exercises (ROM) as soon as the patient initiates C) Initiating (ROM) exercises as soon as possible after the injury D) Performing ROM exercises once a day

Ans: C Passive ROM exercises should be implemented as soon as possible after injury. It would be inappropriate to wait for the patient to first initiate exercises. Toes, metatarsals, ankles, knees, and hips should be put through a full ROM at least four, and ideally five, times daily. Repositioning alone will not prevent contractures.

The nurse is writing a care plan for a patient with brain metastases. The nurse decides that an appropriate nursing diagnosis is anxiety related to lack of control over the health circumstances. In establishing this plan of care for the patient, the nurse should include what intervention? A) The patient will receive antianxiety medications every 4 hours. B) The patients family will be instructed on planning the patients care. C) The patient will be encouraged to verbalize concerns related to the disease and its treatment. D) The patient will begin intensive therapy with the goal of distraction.

Ans: C Patients need the opportunity to exercise some control over their situation. A sense of mastery can be gained as they learn to understand the disease and its treatment and how to deal with their feelings. Distraction and administering medications will not allow the patient to gain control over anxiety. Delegating planning to the family will not help the patient gain a sense of control and autonomy.

The nurse is caring for a patient newly diagnosed with a primary brain tumor. The patient asks the nurse where his tumor came from. What would be the nurses best response? A) Your tumor originated from somewhere outside the CNS. B) Your tumor likely started out in one of your glands. C) Your tumor originated from cells within your brain itself. D) Your tumor is from nerve tissue somewhere in your body.

Ans: C Primary brain tumors originate from cells and structures within the brain. Secondary brain tumors are metastatic tumors that originate somewhere else in the body. The scenario does not indicate that the patients tumor is a pituitary tumor or a neuroma.

The nurse is reviewing the medication administration record of a female patient who possesses numerous risk factors for stroke. Which of the womans medications carries the greatest potential for reducing her risk of stroke? A) Naproxen 250 PO b.i.d. B) Calcium carbonate 1,000 mg PO b.i.d. C) Aspirin 81 mg PO o.d. D) Lorazepam 1 mg SL b.i.d. PRN

Ans: C Research findings suggest that low-dose aspirin may lower the risk of stroke in women who are at risk. Naproxen, lorazepam, and calcium supplements do not have this effect.

The nurse is caring for a patient in the ICU who has a brain stem herniation and who is exhibiting an altered level of consciousness. Monitoring reveals that the patients mean arterial pressure (MAP) is 60 mm Hg with an intracranial pressure (ICP) reading of 5 mm Hg. What is the nurses most appropriate A) Position the patient in the high Fowlers position as tolerated. B) Administer osmotic diuretics as ordered. C) Participate in interventions to increase cerebral perfusion pressure. D) Prepare the patient for craniotomy.

Ans: C The cerebral perfusion pressure (CPP) is 55 mm Hg, which is considered low. The normal CPP is 70 to 100 mm Hg. Patients with a CPP of less than 50 mm Hg experience irreversible neurologic damage. As a result, interventions are necessary. A craniotomy is not directly indicated. Diuretics and increased height of bed would exacerbate the patients condition.

Splints have been ordered for a patient who is at risk of developing footdrop following a spinal cord injury. The nurse caring for this patient knows that the splints are removed and reapplied when? A) At the patients request B) Each morning and evening C) Every 2 hours D) One hour prior to mobility exercises

Ans: C The feet are prone to footdrop; therefore, various types of splints are used to prevent footdrop. When used, the splints are removed and reapplied every 2 hours.

The pathophysiology of an ischemic stroke involves the ischemic cascade, which includes the following steps: 1. Change in pH 2. Blood flow decreases 3. A switch to anaerobic respiration 4. Membrane pumps fail 5. Cells cease to function 6. Lactic acid is generated Put these steps in order in which they occur. A) 635241 B) 352416 C) 236145 D) 162534

Ans: C The ischemic cascade begins when cerebral blood flow decreases to less than 25 mL per 100 g of blood per minute. At this point, neurons are no longer able to maintain aerobic respiration. The mitochondria must then switch to anaerobic respiration, which generates large amounts of lactic acid, causing a change in the pH. This switch to the less efficient anaerobic respiration also renders the neuron incapable of producing sufficient quantities of adenosine triphosphate (ATP) to fuel the depolarization processes. The membrane pumps that maintain electrolyte balances begin to fail, and the cells cease to function.

A patient diagnosed with transient ischemic attacks (TIAs) is scheduled for a carotid endarterectomy. The nurse explains that this procedure will be done for what purpose? A) To decrease cerebral edema B) To prevent seizure activity that is common following a TIA C) To remove atherosclerotic plaques blocking cerebral flow D) To determine the cause of the TIA

Ans: C The main surgical procedure for select patients with TIAs is carotid endarterectomy, the removal of an atherosclerotic plaque or thrombus from the carotid artery to prevent stroke in patients with occlusive disease of the extracranial arteries. An endarterectomy does not decrease cerebral edema, prevent seizure activity, or determine the cause of a TIA.

A patient is being admitted to the neurologic ICU following an acute head injury that has resulted in cerebral edema. When planning this patients care, the nurse would expect to administer what priority medication? A) Hydrochlorothiazide (HydroDIURIL) B) Furosemide (Lasix) C) Mannitol (Osmitrol) D) Spirolactone (Aldactone)

Ans: C The osmotic diuretic mannitol is given to dehydrate the brain tissue and reduce cerebral edema. This drug acts by reducing the volume of brain and extracellular fluid. Spirolactone, furosemide, and hydrochlorothiazide are diuretics that are not typically used in the treatment of increased ICP resulting from cerebral edema.

The nurse is preparing health education for a patient who is being discharged after hospitalization for a hemorrhagic stroke. What content should the nurse include in this education? A) Mild, intermittent seizures can be expected. B) Take ibuprofen for complaints of a serious headache. C) Take antihypertensive medication as ordered. D) Drowsiness is normal for the first week after discharge.

Ans: C The patient and family are provided with information that will enable them to cooperate with the care and restrictions required during the acute phase of hemorrhagic stroke and to prepare the patient to return home. Patient and family teaching includes information about the causes of hemorrhagic stroke and its possible consequences. Symptoms of hydrocephalus include gradual onset of drowsiness and behavioral changes. Hypertension is the most serious risk factor, suggesting that appropriate antihypertensive treatment is essential for a patient being discharged. Seizure activity is not normal; complaints of a serious headache should be reported to the physician before any medication is taken. Drowsiness is not normal or expected.

A patient is recovering from intracranial surgery that was performed using the transsphenoidal approach. The nurse should be aware that the patient may have required surgery on what neurologic structure? A) Cerebellum B) Hypothalamus C) Pituitary gland D) Pineal gland

Ans: C The transsphenoidal approach (through the mouth and nasal sinuses) is often used to gain access to the pituitary gland. This surgical approach would not allow for access to the pineal gland, cerebellum, or hypothalamus.

A neurologic nurse is reviewing seizures with a group of staff nurses. How should this nurse best describe the cause of a seizure? A) Sudden electrolyte changes throughout the brain B) A dysrhythmia in the peripheral nervous system C) A dysrhythmia in the nerve cells in one section of the brain D) Sudden disruptions in the blood flow throughout the brain

Ans: C The underlying cause of a seizure is an electrical disturbance (dysrhythmia) in the nerve cells in one section of the brain; these cells emit abnormal, recurring, uncontrolled electrical discharges. Seizures are not caused by changes in blood flow or electrolytes.

The nurse planning the care of a patient with head injuries is addressing the patients nursing diagnosis of sleep deprivation. What action should the nurse implement? A) Administer a benzodiazepine at bedtime each night. B) Do not disturb the patient between 2200 and 0600. C) Cluster overnight nursing activities to minimize disturbances. D) Ensure that the patient does not sleep during the day.

Ans: C To allow the patient longer times of uninterrupted sleep and rest, the nurse can group nursing care activities so that the patient is disturbed less frequently. However, it is impractical and unsafe to provide no care for an 8 hour period. The use of benzodiazepines should be avoided.

The nurse caring for a patient with a spinal cord injury notes that the patient is exhibiting early signs and symptoms of disuse syndrome. Which of the following is the most appropriate nursing action? A) Limit the amount of assistance provided with ADLs. B) Collaborate with the physical therapist and immobilize the patients extremities temporarily. C) Increase the frequency of ROM exercises. D) Educate the patient about the importance of frequent position changes.

Ans: C To prevent disuse syndrome, ROM exercises must be provided at least four times a day, and care is taken to stretch the Achilles tendon with exercises. The patient is repositioned frequently and is maintained in proper body alignment whether in bed or in a wheelchair. The patient must be repositioned by caregivers, not just taught about repositioning. It is inappropriate to limit assistance for the sole purpose of preventing disuse syndrome.

A patient with a spinal cord injury has experienced several hypotensive episodes. How can the nurse best address the patients risk for orthostatic hypotension? A) Administer an IV bolus of normal saline prior to repositioning. B) Maintain bed rest until normal BP regulation returns. C) Monitor the patients BP before and during position changes. D) Allow the patient to initiate repositioning.

Ans: C To prevent hypotensive episodes, close monitoring of vital signs before and during position changes is essential. Prolonged bed rest carries numerous risks and it is not possible to provide a bolus before each position change. Following the patients lead may or may not help regulate BP.

A nurse is collaborating with the interdisciplinary team to help manage a patients recurrent headaches. What aspect of the patients health history should the nurse identify as a potential contributor to the patients headaches? A) The patient leads a sedentary lifestyle. B) The patient takes vitamin D and calcium supplements. C) The patient takes vasodilators for the treatment of angina. D) The patient has a pattern of weight loss followed by weight gain.

Ans: C Vasodilators are known to contribute to headaches. Weight fluctuations, sedentary lifestyle, and vitamin supplements are not known to have this effect.

A patient who has been experiencing numerous episodes of unexplained headaches and vomiting has subsequently been referred for testing to rule out a brain tumor. What characteristic of the patients vomiting is most consistent with a brain tumor? A) The patients vomiting is accompanied by epistaxis. B) The patients vomiting does not relieve his nausea. C) The patients vomiting is unrelated to food intake. D) The patients emesis is blood-tinged.

Ans: C Vomiting is often unrelated to food intake if caused by a brain tumor. The presence or absence of blood is not related to the possible etiology and vomiting may or may not relieve the patients nausea.

The nurse educator is discussing neoplasms with a group of recent graduates. The educator explains that the effects of neoplasms are caused by the compression and infiltration of normal tissue. The physiologic changes that result can cause what pathophysiologic events? Select all that apply. A) Intracranial hemorrhage B) Infection of cerebrospinal fluid C) Increased ICP D) Focal neurologic signs E) Altered pituitary function

Ans: C, D, E The effects of neoplasms are caused by the compression and infiltration of tissue. A variety of physiologic changes result, causing any or all of the following pathophysiologic events: increased ICP and cerebral edema, seizure activity and focal neurologic signs, hydrocephalus, and altered pituitary function.

A patient who suffered a spinal cord injury is experiencing an exaggerated autonomic response. What aspect of the patients current health status is most likely to have precipitated this event? A) The patient received a blood transfusion. B) The patients analgesia regimen was recent changed. C) The patient was not repositioned during the night shift. D) The patients urinary catheter became occluded.

Ans: D A distended bladder is the most common cause of autonomic dysreflexia. Infrequent positioning is a less likely cause, although pressure ulcers or tactile stimulation can cause it. Changes in mediations or blood transfusions are unlikely causes.

A patient diagnosed with a cerebral aneurysm reports a severe headache to the nurse. What action is a priority for the nurse? A) Sit with the patient for a few minutes. B) Administer an analgesic. C) Inform the nurse-manager. D) Call the physician immediately.

Ans: D A headache may be an indication that the aneurysm is leaking. The nurse should notify the physician immediately. The physician will decide whether administration of an analgesic is indicated. Informing the nurse-manager is not necessary. Sitting with the patient is appropriate, once the physician has been notified of the change in the patients condition.

While assessing the patient at the beginning of the shift, the nurse inspects a surgical dressing covering the operative site after the patients cervical diskectomy. The nurse notes that the drainage is 75% saturated with serosanguineous discharge. What is the nurses most appropriate action? A) Page the physician and report this sign of infection. B) Reinforce the dressing and reassess in 1 to 2 hours. C) Reposition the patient to prevent further hemorrhage. D) Inform the surgeon of the possibility of a dural leak.

Ans: D After a cervical diskectomy, the nurse will monitor the operative site and dressing covering this site. Serosanguineous drainage may indicate a dural leak. This constitutes a risk for meningitis, but is not a direct sign of infection. This should be reported to the surgeon, not just reinforced and observed.

The ED is notified that a 6-year-old is in transit with a suspected brain injury after being struck by a car. The child is unresponsive at this time, but vital signs are within acceptable limits. What will be the primary goal of initial therapy? A) Promoting adequate circulation B) Treating the childs increased ICP C) Assessing secondary brain injury D) Preserving brain homeostasis

Ans: D All therapy is directed toward preserving brain homeostasis and preventing secondary brain injury, which is injury to the brain that occurs after the original traumatic event. The scenario does not indicate the child has increased ICP or a secondary brain injury at this point. Promoting circulation is likely secondary to the broader goal of preserving brain homeostasis.

A nurse in the ICU is providing care for a patient who has been admitted with a hemorrhagic stroke. The nurse is performing frequent neurologic assessments and observes that the patient is becoming progressively more drowsy over the course of the day. What is the nurses best response to this assessment finding? A) Report this finding to the physician as an indication of decreased metabolism. B) Provide more stimulation to the patient and monitor the patient closely. C) Recognize this as the expected clinical course of a hemorrhagic stroke. D) Report this to the physician as a possible sign of clinical deterioration.

Ans: D Alteration in LOC often is the earliest sign of deterioration in a patient with a hemorrhagic stroke. Drowsiness and slight slurring of speech may be early signs that the LOC is deteriorating. This finding is unlikely to be the result of metabolic changes and it is not expected. Stimulating a patient with an acute stroke is usually contraindicated.

A patient with a documented history of seizure disorder experiences a generalized seizure. What nursing action is most appropriate? A) Restrain the patient to prevent injury. B) Open the patients jaws to insert an oral airway. C) Place patient in high Fowlers position. D) Loosen the patients restrictive clothing.

Ans: D An appropriate nursing intervention would include loosening any restrictive clothing on the patient. No attempt should be made to restrain the patient during the seizure because muscular contractions are strong and restraint can produce injury. Do not attempt to pry open jaws that are clenched in a spasm to insert anything. Broken teeth and injury to the lips and tongue may result from such an action. If possible, place the patient on one side with head flexed forward, which allows the tongue to fall forward and facilitates drainage of saliva and mucus.

The nurse is caring for a patient with increased intracranial pressure (ICP). The patient has a nursing diagnosis of ineffective cerebral tissue perfusion. What would be an expected outcome that the nurse would document for this diagnosis? A) Copes with sensory deprivation. B) Registers normal body temperature. C) Pays attention to grooming. D) Obeys commands with appropriate motor responses.

Ans: D An expected outcome of the diagnosis of ineffective cerebral tissue perfusion in a patient with increased intracranial pressure (ICP) would include obeying commands with appropriate motor responses. Vitals signs and neurologic status are assessed every 15 minutes to every hour. Coping with sensory deprivation would relate to the nursing diagnosis of disturbed sensory perception. The outcome of registers normal body temperature relates to the diagnosis of potential for ineffective thermoregulation. Body image disturbance would have a potential outcome of pays attention to grooming.

The nurse is caring for a patient who has undergone supratentorial removal of a pituitary mass. What medication would the nurse expect to administer prophylactically to prevent seizures in this patient? A) Prednisone B) Dexamethasone C) Cafergot D) Phenytoin

Ans: D Antiseizure medication (phenytoin, diazepam) is often prescribed prophylactically for patients who have undergone supratentorial craniotomy because of the high risk of seizures after this procedure. Prednisone and dexamethasone are steroids and do not prevent seizures. Cafergot is used in the treatment of migraines.

The nurse is providing care for a patient who is withdrawing from heavy alcohol use. The nurse and other members of the care team are present at the bedside when the patient has a seizure. In preparation for documenting this clinical event, the nurse should note which of the following? A) The ability of the patient to follow instructions during the seizure. B) The success or failure of the care team to physically restrain the patient. C) The patients ability to explain his seizure during the postictal period. D) The patients activities immediately prior to the seizure.

Ans: D Before and during a seizure, the nurse observes the circumstances before the seizure, including visual, auditory, or olfactory stimuli; tactile stimuli; emotional or psychological disturbances; sleep; and hyperventilation. Communication with the patient is not possible during a seizure and physical restraint is not attempted. The patients ability to explain the seizure is not clinically relevant.

What should the nurse suspect when hourly assessment of urine output on a patient postcraniotomy exhibits a urine output from a catheter of 1,500 mL for two consecutive hours? A) Cushing syndrome B) Syndrome of inappropriate antidiuretic hormone (SIADH) C) Adrenal crisis D) Diabetes insipidus

Ans: D Diabetes insipidus is an abrupt onset of extreme polyuria that commonly occurs in patients after brain surgery. Cushing syndrome is excessive glucocorticoid secretion resulting in sodium and water retention. SIADH is the result of increased secretion of ADH; the patient becomes volume-overloaded, urine output diminishes, and serum sodium concentration becomes dilute. Adrenal crisis is undersecretion of glucocorticoids resulting in profound hypoglycemia, hypovolemia, and hypotension.

The nurse has implemented interventions aimed at facilitating family coping in the care of a patient with a traumatic brain injury. How can the nurse best facilitate family coping? A) Help the family understand that the patient could have died. B) Emphasize the importance of accepting the patients new limitations. C) Have the members of the family plan the patients inpatient care. D) Assist the family in setting appropriate short-term goals.

Ans: D Helpful interventions to facilitate coping include providing family members with accurate and honest information and encouraging them to continue to set well-defined, short-term goals. Stating that a patients condition could be worse downplays their concerns. Emphasizing the importance of acceptance may not necessarily help the family accept the patients condition. Family members cannot normally plan a patients hospital care, although they may contribute to the care in some ways.

A 13-year-old was brought to the ED, unconscious, after being hit in the head by a baseball. When the child regains consciousness, 5 hours after being admitted, he cannot remember the traumatic event. MRI shows no structural sign of injury. What injury would the nurse suspect the patient has? A) Diffuse axonal injury B) Grade 1 concussion with frontal lobe involvement C) Contusion D) Grade 3 concussion with temporal lobe involvement

Ans: D In a grade 3 concussion there is a loss of consciousness lasting from seconds to minutes. Temporal lobe involvement results in amnesia. Frontal lobe involvement can cause uncharacteristic behavior and a grade 1 concussion does not involve loss of consciousness. Diagnostic studies may show no apparent structural sign of injury, but the duration of unconsciousness is an indicator of the severity of the concussion. Diffuse axonal injury (DAI) results from widespread shearing and rotational forces that produce damage throughout the brainto axons in the cerebral hemispheres, corpus callosum, and brain stem. In cerebral contusion, a moderate to severe head injury, the brain is bruised and damaged in a specific area because of severe acceleration-deceleration force or blunt trauma.

A patient has just been diagnosed with Parkinsons disease and the nurse is planning the patients subsequent care for the home setting. What nursing diagnosis should the nurse address when educating the patients family? A) Risk for infection B) Impaired spontaneous ventilation C) Unilateral neglect D) Risk for injury

Ans: D Individuals with Parkinsons disease face a significant risk for injury related to the effects of dyskinesia. Unilateral neglect is not characteristic of the disease, which affects both sides of the body. Parkinsons disease does not directly constitute a risk for infection or impaired respiration.

A patient has a poor prognosis after being involved in a motor vehicle accident resulting in a head injury. As the patients ICP increases and condition worsens, the nurse knows to assess for indications of approaching death. These indications include which of the following? A) Hemiplegia B) Dry mucous membranes C) Signs of internal bleeding D) Loss of brain stem reflexes

Ans: D Loss of brain stem reflexes, including pupillary, corneal, gag, and swallowing reflexes, is an ominous sign of approaching death. Dry mucous membranes, hemiplegia, and bleeding must be promptly addressed, but none of these is a common sign of impending death.

A patient, diagnosed with cancer of the lung, has just been told he has metastases to the brain. What change in health status would the nurse attribute to the patients metastatic brain disease? A) Chronic pain B) Respiratory distress C) Fixed pupils D) Personality changes

Ans: D Neurologic signs and symptoms include headache, gait disturbances, visual impairment, personality changes, altered mentation (memory loss and confusion), focal weakness, paralysis, aphasia, and seizures. Pain, respiratory distress, and fixed pupils are not among the more common neurologic signs and symptoms of metastatic brain disease.

The nurse is caring for a patient with Huntington disease who has been admitted to the hospital for treatment of malnutrition. What independent nursing action should be implemented in the patients plan of care? A) Firmly redirect the patients head when feeding. B) Administer phenothiazines after each meal as ordered. C) Encourage the patient to keep his or her feeding area clean. D) Apply deep, gentle pressure around the patients mouth to aid swallowing.

Ans: D Nursing interventions for a patient who has inadequate nutritional intake should include the following: Apply deep gentle pressure around the patients mouth to assist with swallowing, and administer phenothiazines prior to the patients meal as ordered. The nurse should disregard the mess of the feeding area and treat the person with dignity. Stiffness and turning away by the patient during feeding are uncontrollable choreiform movements and should not be interrupted.

A nurse on the neurologic unit is providing care for a patient who has spinal cord injury at the level of C4. When planning the patients care, what aspect of the patients neurologic and functional status should the nurse consider? A) The patient will be unable to use a wheelchair. B) The patient will be unable to swallow food. C) The patient will be continent of urine, but incontinent of bowel. D) The patient will require full assistance for all aspects of elimination.

Ans: D Patients with a lesion at C4 are fully dependent for elimination. The patient is dependent for feeding, but is able to swallow. The patient will be capable of using an electric wheelchair.

During the examination of an unconscious patient, the nurse observes that the patients pupils are fixed and dilated. What is the most plausible clinical significance of the nurses finding? A) It suggests onset of metabolic problems. B) It indicates paralysis on the right side of the body. C) It indicates paralysis of cranial nerve X. D) It indicates an injury at the midbrain level.

Ans: D Pupils that are fixed and dilated indicate injury at the midbrain level. This finding is not suggestive of unilateral paralysis, metabolic deficits, or damage to CN X.

The public health nurse is planning a health promotion campaign that reflects current epidemiologic trends. The nurse should know that hemorrhagic stroke currently accounts for what percentage of total strokes in the United States? A) 43% B) 33% C) 23% D) 13%

Ans: D Strokes can be divided into two major categories: ischemic (87%), in which vascular occlusion and significant hypoperfusion occur, and hemorrhagic (13%), in which there is extravasation of blood into the brain or subarachnoid space.

A 37-year-old man is brought to the clinic by his wife because he is experiencing loss of motor function and sensation. The physician suspects the patient has a spinal cord tumor and hospitalizes him for diagnostic testing. In light of the need to diagnose spinal cord compression from a tumor, the nurse will most likely prepare the patient for what test? A) Anterior-posterior x-ray B) Ultrasound C) Lumbar puncture D) MRI

Ans: D The MRI scan is the most commonly used diagnostic procedure. It is the most sensitive diagnostic tool that is particularly helpful in detecting epidural spinal cord compression and vertebral bone metastases.

What should be included in the patients care plan when establishing an exercise program for a patient affected by a stroke? A) Schedule passive range of motion every other day. B) Keep activity limited, as the patient may be over stimulated. C) Have the patient perform active range-of-motion (ROM) exercises once a day. D) Exercise the affected extremities passively four or five times a day.

Ans: D The affected extremities are exercised passively and put through a full ROM four or five times a day to maintain joint mobility, regain motor control, prevent development of a contracture in the paralyzed extremity, prevent further deterioration of the neuromuscular system, and enhance circulation. Active ROM exercises should ideally be performed more than once per day.

A patient who was diagnosed with Parkinsons disease several months ago recently began treatment with levodopa-carbidopa. The patient and his family are excited that he has experienced significant symptom relief. The nurse should be aware of what implication of the patients medication regimen? A) The patient is in a honeymoon period when adverse effects of levodopa-carbidopa are not yet evident. B) Benefits of levodopa-carbidopa do not peak until 6 to 9 months after the initiation of treatment. C) The patients temporary improvement in status is likely unrelated to levodopa-carbidopa. D) Benefits of levodopa-carbidopa often diminish after 1 or 2 years of treatment.

Ans: D The beneficial effects of levodopa therapy are most pronounced in the first year or two of treatment. Benefits begin to wane and adverse effects become more severe over time. However, a honeymoon period of treatment is not known.

An ED nurse has just received a call from EMS that they are transporting a 17-year-old man who has just sustained a spinal cord injury (SCI). The nurse recognizes that the most common cause of this type of injury is what? A) Sports-related injuries B) Acts of violence C) Injuries due to a fall D) Motor vehicle accidents

Ans: D The most common causes of SCIs are motor vehicle crashes (46%), falls (22%), violence (16%), and sports (12%).

A male patient with a metastatic brain tumor is having a generalized seizure and begins vomiting. What should the nurse do first? A) Perform oral suctioning. B) Page the physician. C) Insert a tongue depressor into the patients mouth. D) Turn the patient on his side.

Ans: D The nurses first response should be to place the patient on his side to prevent him from aspirating emesis. Inserting something into the seizing patients mouth is no longer part of a seizure protocol. Obtaining supplies to suction the patient would be a delegated task. Paging or calling the physician would only be necessary if this is the patients first seizure.

A patient with a cerebral aneurysm exhibits signs and symptoms of an increase in intracranial pressure (ICP). What nursing intervention would be most appropriate for this patient? A) Range-of-motion exercises to prevent contractures B) Encouraging independence with ADLs to promote recovery C) Early initiation of physical therapy D) Absolute bed rest in a quiet, nonstimulating environment

Ans: D The patient is placed on immediate and absolute bed rest in a quiet, nonstressful environment because activity, pain, and anxiety elevate BP, which increases the risk for bleeding. Visitors are restricted. The nurse administers all personal care. The patient is fed and bathed to prevent any exertion that might raise BP.

After a subarachnoid hemorrhage, the patients laboratory results indicate a serum sodium level of less than 126 mEq/L. What is the nurses most appropriate action? A) Administer a bolus of normal saline as ordered. B) Prepare the patient for thrombolytic therapy as ordered. C) Facilitate testing for hypothalamic dysfunction. D) Prepare to administer 3% NaCl by IV as ordered.

Ans: D The patient may be experiencing syndrome of inappropriate antidiuretic hormone (SIADH) or cerebral salt-wasting syndrome. The treatment most often is the use of IV hypertonic 3% saline. A normal saline bolus would exacerbate the problem and there is no indication for tests of hypothalamic function or thrombolytic therapy.

A nursing student is writing a care plan for a newly admitted patient who has been diagnosed with a stroke. What major nursing diagnosis should most likely be included in the patients plan of care? A) Adult failure to thrive B) Post-trauma syndrome C) Hyperthermia D) Disturbed sensory perception

Ans: D The patient who has experienced a stroke is at a high risk for disturbed sensory perception. Stroke is associated with multiple other nursing diagnoses, but hyperthermia, adult failure to thrive, and posttrauma syndrome are not among these.

A female patient is diagnosed with a right-sided stroke. The patient is now experiencing hemianopsia. How might the nurse help the patient manage her potential sensory and perceptional difficulties? A) Keep the lighting in the patients room low. B) Place the patients clock on the affected side. C) Approach the patient on the side where vision is impaired. D) Place the patients extremities where she can see them.

Ans: D The patient with homonymous hemianopsia (loss of half of the visual field) turns away from the affected side of the body and tends to neglect that side and the space on that side; this is called amorphosynthesis. In such instances, the patient cannot see food on half of the tray, and only half of the room is visible. It is important for the nurse to remind the patient constantly of the other side of the body, to maintain alignment of the extremities, and if possible, to place the extremities where the patient can see them. Patients with a decreased field of vision should be approached on the side where visual perception is intact. All visual stimuli (clock, calendar, and television) should be placed on this side. The patient can be taught to turn the head in the direction of the defective visual field to compensate for this loss. Increasing the natural or artificial lighting in the room and providing eyeglasses are important in increasing vision. There is no reason to keep the lights dim.


Set pelajaran terkait

section 20 unit 1: Real Estate Ethics

View Set

AP Computer Science midterm study guide

View Set

Social Studies Review For Test 1/31/17

View Set

Chapter 1 - Nurse's Role in Health Assessment: Collecting & Analyzing Data

View Set

AP Human Geography Chapter 9 Agriculture

View Set

Chapter 11: Principle of Superposition, Principle of Original Horizontality, Principle of Lateral Continuity, Principle of Cross-Cutting Relationships, Principle of Inclusions

View Set

BUSMHR Exam 1 - Maintenance Roles VS Task Roles

View Set